Constitutional Liberties

From wikilawschool.net. Wiki Law School does not provide legal advice. For educational purposes only.
Constitutional Liberties
Relevant texts Image of American Constitutional Law: Powers and Liberties [Connected eBook with Study Center] (Aspen Casebooks)
American Constitutional Law: Powers and Liberties [Connected eBook with Study Center] (Aspen Casebooks)


Image of Constitutional Law for a Changing America: Rights, Liberties, and Justice
Constitutional Law for a Changing America: Rights, Liberties, and Justice
Image of Constitutional Analysis in a Nutshell (Nutshells)
Constitutional Analysis in a Nutshell (Nutshells)

Related course(s) Constitutional Law

What is Speech?[edit | edit source]

People often communicate through symbols other than words. Marches, picketing, armbands are just a few examples of expressive conduct. To deny 1st Amendment protection for such forms of communication would mean a loss of some of the most effective means of communicating messages.

Thus, the Supreme Court long has protected conduct that communicates under the first amendment. For example, the Supreme Court invalidated a law that required students to salute the flag. The Court explained that symbolism is a primitive but effective way of expression, the use of a flag to symbolize some system, idea, or institution is a shortcut form mind to mind.

Conduct of all sorts can convey a message. Yet if taken to the extreme, it would mean virtually every criminal law would have to meet strict scrutiny.

What Conduct is Communicative[edit | edit source]

The Court observed that it is possible to find some kernel of expression in almost every activity a person does. But such kernel is not sufficient to bring the activity within the protection of the first amendment. In other words, under this approach, conduct is analyzed as speech under the first amendment if, there is the intent to convey a specific message and second, there is substantial likelihood that the message would be understood by viewers.  

Levels of Scrutiny[edit | edit source]

  • What is the end that the government is trying to achieve? What is the government trying to accomplish?
  • What are the means the government is using? How is the government trying to accomplish those ends?

Strict Scrutiny[edit | edit source]

  • Applies when the statute or policy at issue infringes a fundamental right
  • Burden of proof is on the government seeking to uphold the law
  • End test – does the statute further a compelling governmental purpose? If not, then it fails.
  • Means test – are the means necessary to achieve the compelling purpose?
    1. Do the means actually achieve the compelling purpose?
    2. Are the means no-less-rights-restrictive than necessary to achieve the purpose?

Rational Basis review[edit | edit source]

  • If no fundamental right is being infringed
    1. Burden of proof is on the party challenging the policy or statute
    2. End test – government interests only needs to be legitimate and not compelling
    3. Means test- means chosen only need to rationally relate to the government’s purpose
      1. Really easy test to pass
      2. Rationally related is interpreted as a “non-crazy way”
      3. Purpose does not even have to be the real purpose – it can be anything the Court thinks up

Intermediate Scrutiny[edit | edit source]

  • Burden of proof is still on the government
  • Ends test- government’s end only needs to be important (easier than compelling, harder than legitimate)
  • Means test – government’s means need to be substantially related to achieving the government’s ends (easier than necessary, harder than rationally related)
  • When the Court says “apply a heightened level of scrutiny” it can mean this or strict.

Analytical Framework to use scrutiny[edit | edit source]

Step 1[edit | edit source]

  • Is speech being regulated?
    1. If no then not protected by the first amendment
    2. If yes
      1. Does this fall under government speech? If yes, use that separate analytical approach
      2. If this does not fall into government speech, go to step 2

Step 2[edit | edit source]

  • Is the speech being restricted or compelled in a serious way? (infringement = a direct ban or compulsion of expression, or a direct and substantial interference with expression)
    1. If no, ten rational basis review applies
    2. If yes, then step 3

Step 3[edit | edit source]

  • Is the regulation unconstitutionally vague and/or substantially overbroad?
    1. If the answer is Yes, then it is unconstitutional, but keep doing the analysis in case the Court disagrees with you
    2. If no, go to next step
    3. Note, all laws are somewhat vague or overbroad. The question is whether the law or policy is too vague or too overbroad. This is not a strict scrutiny issue, it is automatically unconstitutional if it is too vague and too overbroad

Step 4[edit | edit source]

  • Is the regulation content based?
    1. If no, it is content-neutral, which means it gets intermediate scrutiny
      1. Under IS, to be upheld, the government must have an important interest in regulating the speech and means chosen must be substantially related to that interest
      2. Time/Place/Manner version: Is the law narrowly tailored or closely related to serving a significant or substantial government interest? Also examine the availability of alternative channels of communication
    2. If yes, it is content-based, go to next step

Step 5[edit | edit source]

  • Is speech in one of the not-fully-protected content-based categories? (incitement of fighting words, obscenity, child porn, some student speech, libel)
    1. If yes, then it gets a rational basis review
      1. But it may be the subject to its own particular set of tests established by the Court
      2. But if government regulates SOME speech in the category and not the other speech, based on content, the regulation gets strict scrutiny
    2. If No, it gets strict scrutiny
      1. Unconstitutional unless government has compelling interest in regulating speech and the means chosen are necessary to achieve that purpose
      2. Important: If the law is both content based and viewpoint based, it will fail strict scrutiny because the government interest is illegitimate – as there is no compelling interest for government censorship based on viewpoint

State Action[edit | edit source]

Generally, the Congress may not regulate the actions of private entities. Often for cases involving the Constitution, the first question to answer is whether there is a state actor involved. State actors can be State government officials, judges, and Congress itself. However, it can also extend to people the State has given authority to such as public school teachers.

Another factor to consider is whether an entity is truly a private one. In Marsh v Alabama, the Court held that a company-owned town is considered a public entity if it is open to the public. Another consideration is to see whether the action of the private entity is in a close enough nexus between the state and the private action.

Private entities as public/state actors[edit | edit source]

MANHATTAN COMMUNITY ACCESS CORPORATION v. HALLECK[edit | edit source]

Facts: The plaintiffs made a film which they broadcasted on MNN (a public access network) and it led to their suspension due to its controversial nature. They alleged that this was a violation of their first amendment rights.

Rules: The public-function exception to the state-action doctrine considers a private entity a governmental actor only if it is performing a traditionally and exclusively public function.

Reasoning: To preserve robust protection of individual liberty, private entities are treated as governmental actors only in limited circumstances. Private entities may be considered governmental actors if they are compelled by the government to perform a particular action, if the act jointly with the government, and if publication function exception to state-action doctrine applies. Merely hosting a platform for speech doesn’t transform a private entity into a governmental actor subject to the first amendment. Very few meet this category.

Marsh v. Alabama[edit | edit source]

Facts: A JHV’s witness was charged with trespassing for spreading pamphlets in the company-town of Chickasaw. She was told she needed a permit and was told to leave. When she asked for a permit, she was kicked out. She tried again to spread her pamphlets, but was arrested.

Rules: The first and fourteenth amendment protections of speech and religion still apply to individuals when operating in a privately-owned town if the town is open to the public.

Reasoning: Chickasaw acted like a regular municipality and permitted non-employees to enter its facilities. Thus, Marsh had the right to distribute her religious pamphlets there.

Had the title to Chickasaw belonged not to a private but to a municipal corporation and had appellant been arrested for violating a municipal ordinance rather than a ruling by those appointed by the corporation to manage a company-town it would have been clear that appellant’s conviction must be reversed.

Jackson v. Metropolitan Edison Co.[edit | edit source]

Facts: A guy named Dobson placed the electric bill under his name and never paid. Jackson’s electricity was cut because of this, she requested that the new account is to be made under her son’s name, which the company did not do so since he’s only 12. The electricity was cut off. Jackson sued Metropolitan, alleging that this was an unconstitutional taking of property since Metropolitan had a monopoly, was state regulated, and had a public interest and is thus a state actor.

Rules: For the purposes of the fourteenth amendment, an action of a private entity will only be treated as a state action if there is a sufficiently close nexus between the state and the challenged action of the private entity so that the action of the latter may be fairly treated as the state itself.

Reasoning: Even if a business has a public-interest, has a monopoly, or state-regulated, it still would not be considered state action. A business is only considered a state actor if its actions are so connected with the state that it is essentially the actions of the state. The state is not required to provide electricity, so Metropolitan’s actions was not subject to state regulation.

Perhaps in recognition of the fact that the supplying of utility service is not traditionally the exclusive prerogative of the State, petitioner invites the expansion of the doctrine of this limited line of cases into a broad principle that all businesses “affected with the public interest” are state actors in all their actions. We decline the invitation.

Marsh and Jackson use very different tests for public function. Marsh uses a balancing test and looks to whether the private property is used for a public purpose. Jackson focuses on whether it is an activity traditionally, exclusively done by the government.

Application of the Close Nexus rule[edit | edit source]

Terry v Adams[edit | edit source]

Facts: Fort Bend County was using the guise of a political party, through the Jaybird Party, to influence elections and to discriminate against black voters and candidates.

Rules: The Fifteenth Amendment does not permit the exclusion of African-American voters from primary elections run by private parties when those elections ultimately influences later publicly-run elections.

Reasoning: The county was using the Jaybird Party as a device to produce the equivalent of a prohibited election by circumventing the fifteenth amendment. It acted unconstitutionally by doing so. The fifteenth amendment bans all racial discrimination in voting, affects all level of government, and applies to private political clubs.

  • Here the Jaybird club influenced the election heavily in order to discriminate against the black community. The actor is the club and the challenged act is their discrimination. A close nexus that bridges them is the results of the election.

Smith v Allwright[edit | edit source]

Facts: Texas mandated that its political parties hold primary elections. Moreover, Texas delegated the primary election process to the parties themselves. Only party members could vote in a primary election.

Rule: States may not permit parties to restrict primary voting on a racial basis.

Reasoning: According to the Fifteenth Amendment, such political parties perform a state function, and the state holds responsibility for the decision. Direct action by states to restrict primary voting access on a racial basis violates equal protection.

Evans v. Newton[edit | edit source]

Facts: In 1911, United States Senator Augustus Bacon executed a will devising a tract of land to the Mayor and City Council of the City of Macon, Georgia. The will specifically provided that the land was to be used as a public park open exclusively to white people and managed by white people. The City of Macon managed the park through its Board of Managers according to these terms for a while, but then opened it up to African Americans when it believed it could no longer constitutionally exclude them.

Rules: If a gift or trust from a private individual to a public entity contains a requirement that property be segregated by race or color, the public entity may not enforce the segregation.

Reasoning: The city remained entwined in the park’s management and control. The park thus remained subject to the restraints of the Fourteenth Amendment. Merely changing the trustee from a public entity to a private entity will not change the “public” character of the park. This conclusion is supported by the nature of the service, as parks are traditionally thought of as public areas that are open to and used by the public.

One is the right of the individual to pick his own associates so as to express his preferences and dislikes, and to fashion his private life by joining such clubs and groups as he chooses. The other is the constitutional ban in the Equal Protection Clause of the Fourteenth Amendment against state-sponsored racial inequality, which of course bars a city from acting as trustee under a private will that serves the racial segregation cause.

What is “private” action and what is “state” action is not always easy to determine. “Only by sifting facts and weighing circumstances” can we determine whether the reach of the Fourteenth Amendment extends to a particular case. The range of government activities is broad and varied, and the fact that government has engaged in a particular activity does not necessarily mean that an individual entrepreneur or manager of the same kind of undertaking suffers the same constitutional inhibitions.

Hudgens v National Labor Relations Board[edit | edit source]

Facts: Hudgens’ shopping center had a Butler Shoe branch. Employees of Butler Shoe all protested via a strike. The employees picketed in front of the Hudgens Branch of Butler Shoe and were asked to leave.

Rules: A private shopping mall may constitutionally exclude picketing on its premises even if that picketing relates to the actual activities of the tenant stores.

Reasoning: The Constitution’s free-speech guarantee guards against only government regulation of speech. Private parties are typically free to control speech on their property. Some exceptions to exists if a private property acts like public property like in Marsh

Logan Valley Holding - Where a privately-owned shopping center serves as the community business block and is open to the general public, the state may not use its “no trespassing” laws to prevent citizens from exercising their First Amendment rights to peacefully picket on its premises.

Lloyd Holding – A private business may constitutionally exclude the distribution of handbills on its property when those handbills are completely unrelated to the business’ functions, and there are alternative means for distributors to relay their message.

Both are overruled by this case.

Entanglement Exception[edit | edit source]

Under this exception, the Constitution applies if the government affirmatively authorizes, encourages, or facilitates private conduct that violates the Constitution. Either the government must cease its involvement with the private actor or the private entity must comply with the Constitution.

The key question, then, is what degree of government involvement is sufficient to make the Constitution applicable? What type of government encouragement are sufficient for state action? Unfortunately, the entanglement exception cases are even more inconsistent than those concerning the public function exception.

Judicial and Law Enforcement actions[edit | edit source]

Shelley v. Kraemer[edit | edit source]

Facts: The Shelleys bought a house in a racist area that had an anti-black anti-asian covenant, which the homeowners sought to enforce.

Rules: State court enforcement of a racially restrictive covenant constitutes state action that violates the Equal protection clause of the 14th amendment.

Reasoning: The covenant denied the Shelleys rights guaranteed by the 14th amendment when it didn’t give them an equal protection as other races would, a due process on the takings of their property, nor privileges and immunities from the enforcement of the covenant. The private nature of the covenant was not unconstitutional. However, the judicial enforcement of it is considered a state action, and that state action is unconstitutional.

The short of the matter is that from the time of the adoption of the Fourteenth Amendment until the present, it has been the consistent ruling of this Court that the action of the States to which the Amendment has reference, includes action of state courts and state judicial officials.

Fair Attribution[edit | edit source]

Lugar v. Edmondson Oil Co.[edit | edit source]

Facts: Lugar (defendant) owned and operated a truck stop. He became indebted to its supplier, Edmondson Oil Co. (plaintiff). Edmonson sued for the debt in Virginia state court, and sought a prejudgment attachment of Lugar's property. This required Edmonson to allege a reasonable belief that Lugar was disposing of property to keep it from creditors.

Rules: Due-process requirements must be satisfied in prejudgment attachment proceedings if state officials jointly participate with private creditors in securing the property in dispute.

Reasoning: Is the depravation of a federal right fairly attributable to the state?

  • The claimed deprivation must result from the exercise of some right or privilege having its source in state authority
  • The party accused of the depravation must be someone who can fairly be treated as a state actor.
    • This includes state officials or anyone who act as or gets reasonable aid from the state to finish the deprivation.

Here, the claimed deprivation resulted from the exercise of some right, which is the filing of the prejudgment attachment through the clerk. The use of the clerk and sheriff also satisfies the second element of being fairly treated as a state actor.

The Court of Appeals erred in holding that in this context “joint participation” required something more than invoking the aid of state officials to take advantage of state-created attachment procedures.

Edmonson v. Leesville Concrete Co.[edit | edit source]

  1. any court of the United States, upon the filing of an appropriate pleading, may declare the rights and other legal relations of any interested party seeking such declaration, whether or not further relief is or could be sought. Any such declaration shall have the force and effect of a final judgment or decree and shall be reviewable as such.

Issues: Does a private civil litigant violate the constitutional principle of equal protection by using preemptory challenges to remove prospective jurors based on their race?

Rules: A private litigant in a civil case may not use preemptory challenges to exclude jurors on account of their race because the exercise or preemptory challenges invokes state action. Race-based preemptory challenges by private litigants violate equal protection.

Holding: Yes

Reasoning:  Although a private civil litigant’s conduct doesn’t implicate constitutional protections.

Can the private party charged with the deprivation be described in all fairness as a state actor? Look at:

  • Extent to which the actor relies on governmental assistance and benefits
  • Whether the actor is performing a traditional government function
  • Whether the injury caused is aggravated in a unique way by the incidents of

governmental authority

Thanks to Lugar we ask first whether the claimed constitutional deprivation resulted from the exercise of a right or privilege having its source in state authority and second, whether the private party charged with deprivation could be described in all fairness as a state actor.

The first factor is satisfied because the challenges were from an exercise of a right having it source in state authority. Second, Leesvile in this case functioned as a government actor, and because the court allowed for this race-based action by a government actor, the constitutional harm was aggravated because it permitted such action in the halls of justice.

The Supreme Court ruled that under the equal protection component of the due process clause of the Fifth Amendment, a private civil litigant that originated its case in a federal district court could not use preemptory challenges to exclude prospective jurors on the account of race. These challenges were only permitted when a statute or law allowed them.

Government Regulation[edit | edit source]

The court also has considered the entanglement exception in instances when the government licenses or regulates an activity. In general, government licensing or regulating is insufficient for finding of a state action, unless there is other government encouraging or facilitating of unconstitutional conduct.

Burton v Wilmington Parking Authority[edit | edit source]

Facts: The authority owned land which it leased to Eagle Coffee. Burton tried to buy a drink from the shop, but was told to leave because he wasn’t white. Burton sued.

Rules: If government action is sufficiently intertwined with private commerce, the private actors’ behavior may be governed by the Equal Protection clause.

Reasoning: If a state or local government participates in private enterprise, then the interplay may constitute enough state action for the 14th amendment to govern the party’s action. The Parking authority owned the property and it is used for their public function of creating parking spaces. The leased areas for the coffee shop was a commercial activity to make the structure economically self-sustaining. The parking authority and the coffee shop were intertwined, and therefore, the coffee shop can be governed by the Equal Protection Clause.

It is impossible to create a precise formula or rule. Only after sifting facts and circumstances can the extent of state involvement in private conduct can be evaluated.

Moose Lodge No. 107 v. Irvis[edit | edit source]

Facts: Moose Lodge was a whites only club and Irvis was invited to come in, but was refused service by staff.

Rules: The issuance of a state license to a private organization did not turn it into a state actor.

Reasoning: The lodge was a private actor that received no state funding and Irvis was not invited to be nor was he ever a member of the lodge, so he lacked standing to sue about membership discrimination and can only do so relating to guest discriminations. In that regard, nothing that the lodge did can be fairly attributed to a state actor.

There was a lack of a more active involvement by the government in the entity’s affairs nor was there a relationship between the private entity and the government.

Entwinement[edit | edit source]

The Supreme Court has recognized two narrow exceptions to the state action requirement: the public functions exception, which provides that private conduct must comply with the constitution when it is performing a task that has traditionally, exclusively, been done by the government; and entanglement, in which the government affirmatively authorizes, encourages, or facilitates unconstitutional conduct.

BRENTWOOD ACADEMY v. TENNESSEE SECONDARY SCHOOL ATHLETIC ASSOCIATION[edit | edit source]

Rules: A private statewide regulatory association with a majority of its membership as public entities engages in state action if it enforces rules against a member.

Holding: Yes.

Reasoning: State action occurs if there’s a close nexus between the state and the action in question. In this case, the Court looked at the facts on whether there was pervasive public entwinement in an athletic association’s management or control and whether the association’s member public schools are from a single state. Pervasive government entwinement occurred because 84% of the members were public school and all the schools are from one state. Therefore, the association engaged in state action.

Brentwood Rule

A private actor’s challenged activity can be state action if:

  • The activity results from the state’s exercise of coercive power
  • The state overtly or covertly provides significant encouragement
  • The private actor willfully participates in joint activity with the state or its agents.

First Amendment – Freedom of Expression[edit | edit source]

Types of Protected Speech[edit | edit source]

The Supreme Court has identified some categories of unprotected speech that the government can prohibit and punish. Incitement of illegal activity, fighting words, and obscenity are such examples. Additionally, there are categories of less-protected speech where the government has more latitude of regulate than usual under the First Amendment. For instance, government generally can regulate commercial speech if intermediate scrutiny is met. Also, the Court has indicated that some types of sexually oriented speech, although protected by the first Amendment, are deemed law value and thus susceptible to government regulation.

These categories are defined based on the subject matter of the speech and thus represent an exception to the usual rule that content-based regulation must meet strict scrutiny. It was traditionally thought that the government had broad latitude to prohibit ad regulate speech within these categories of unprotected expression. The conventional view was that laws in these areas would be upheld as long as they met the rational basis test that all government actions must satisfy. However, the Court indicated that generally, content-based distinctions within categories of unprotected speech must meet strict scrutiny.

Incitement of Illegal Activities[edit | edit source]

The issue of incitement also is important because it poses a basic value question: How should society balance its need for social order against its desire to protect freedom of speech? When, if at all, may speech that advocates criminal activity or the overthrow of the government be stopped to promote order and security?

Some commentators have argued that all such advocacy of illegal conduct should be deemed unprotected by the first amendment. The supreme court has never taken this view.

Thus, the court has been confronted with the task of defining when advocacy of illegality constitutes unprotected incitement and when it is safeguarded by the First Amendment. Over the court of this century, the Supreme Court has used at least four major approaches in this area.

The clear and present danger test[edit | edit source]

Schenk v United States[edit | edit source]

Facts: Schenk was distributing leaflets which encouraged people to oppose the draft during World War I. They were charged with the violation of the Espionage Act of 1917 which outlawed the obstruction of military recruiting and enlisting.

Rules: During wartime, the distribution of written material urging peaceful resistance by making statements against the draft’s legality poses a clear and present danger no protected by the first amendment.

Reasoning: The distribution of leaflets wasn’t speech or expressive conducted protected by the first amendment. If the country hadn’t been at war, the defendant’s expression would’ve been protected by the First Amendment, however, duh, it was at war. In a world war. Also, they specifically targeted their words at draft-eligible men with the intent to obstruct the draft. The first amendment doesn’t protect speech or expressive conduct that creates a clear and present danger of: (1) causing others to violate the law and (2) causing harm to others.

FROHWERK v. UNITED STATES[edit | edit source]

Facts: Two months after the United States’ entry into World War I, Congress enacted the Espionage Act of 1917 (EA). The law made it a crime for any person during time of war, to “willfully cause or attempt to cause insubordination, disloyalty, mutiny, or refusal of duty in the military or the naval forces of the United States.” In 1915, the Missouri Staats Zeiung, a newspaper published in Kansas City, Missouri, issued a series of twelve articles written by Jacob Frohwerk (defendant) denouncing the United States’ involvement in World War I

Rules: The First Amendment’s protection for freedom of speech doesn’t render all types of speech immune from legislative constraints.

Reasoning: In Schenk, the court held that the First Amendment doesn’t protect a person’s words if there is a clear and present danger that they will bring about the substantive evils that Congress has the right to prevent. Here, the clear and present danger rule requires that the speech be reviewed in the context when and where it was made and if it is likely to incite unlawful conduct.

Debs v. United States[edit | edit source]

Facts: The Espionage Act of 1917 (EA) made it a crime to “convey information with intent to interfere with the operation or success of the United States or to promote the success of its enemies.” The EA had the effect of constraining sedition and political speech during World War I. In 1918, Eugene Debs (defendant), leader of the Socialist Party of America, gave a speech protesting the United States’ involvement in World War I

Rules: Speech critical of the United States government’s war effort may be punished as interference with a war effort in time of war

Reasoning: While the main theme of Debs’ speech was to promote the socialist movement, it also encouraged the resistance of the draft.

Abrams v. United States[edit | edit source]

Facts: Abrams and four others (plaintiffs) were convicted of conspiring to violate the Espionage Act of 1917 (EA), as amended in 1918. Abrams printed many copies of leaflets, written both in English and Yiddish, denouncing the United States’ decision to send troops to Russia as part of World War I. Other leaflets denounced the United States’ general involvement in World War I and United States’ efforts to curtail the Russian Revolution.

Rules: Speech that would ordinarily be protected by the First Amendment, may nevertheless be prohibited when it is used in such circumstances and is of such a nature that as to create a clear and present danger of substantive evils that Congress has a right to prevent

Reasoning: The leaflets are inciting sedition at the height of war.

Holmes Dissent: Only speech that poses a present danger of immediate evil can be restricted. These leaflets do not pose any immediate danger to the United States. “The ultimate good desired is better reached by free trade in ideas”

The Reasonableness Approach[edit | edit source]

Gitlow v New York[edit | edit source]

Facts: Gitlow, a socialist, was arrested in 1919 for distributing a “Left Wing Manifesto" that called for the establishment of socialism through strikes and class action of any form. Gitlow was convicted under New York’s Criminal Anarchy Law, which punished advocating the overthrow of the government by force.

Rules: Producing and distributing written materials advocating for the government’s overthrow is a clear and present danger that a state could criminally punish.

Reasoning: The state didn’t violate the First Amendment by punishing Gitlow for producing and distributing the manifesto because he posed a clear and present danger. The court assumes that Freedom of Speech and the press apply to both state governments and federal governments via the Due Process Clause. The New York criminal anarchy statute didn’t violate Gitlow’s right to free speech. The manifesto posed a clear and present danger, an exception to the first amendment. The manifesto wasn’t an abstract statement of political belief. It advocated and urged action to overthrow and destroy democracy and capitalism. The natural tendency and probable effect of the manifesto was to bring about the evil that the state was entitled to prevent.

Whitney v. California[edit | edit source]

Facts: California’s Criminal Syndicalism Act (CCSA) prohibited “advocating, teaching, or aiding and abetting the commission of crime, sabotage, or unlawful acts of force and violence or unlawful methods of terrorism as a means of accomplishing a change in the industrial ownership or control or effecting any political change.”

Rules: The first amendment allows states to criminalize membership to an organization that advocates the illegal overthrow of government.

Reasoning: The first amendment didn’t protect the type of advocacy in which Whitney and her fellow communist party members engaged at the state convention. Her organization makes statements that endangers the foundations of organized government.

The Brandenburg Test[edit | edit source]

Brandenburg v. Ohio[edit | edit source]

Facts: Brandenburg (defendant) was a leader of the Ku Klux Klan in the State of Ohio (plaintiff). Brandenburg was convicted under the Ohio Criminal Syndicalism Act (OCSA) for “advocating the duty, necessity, or propriety of crime, sabotage, violence, or unlawful methods of terrorism as a means of accomplishing industrial or political reform,” and for “voluntarily assembling with any society, group, or assemblage of persons formed to teach or advocate the doctrines of criminal syndicalism.”

Rules: Under the first amendment, a state may not outlaw speech unless the speech is directed to inciting violence and imminently likely to do so.

Reasoning: The court here distinguished the difference between abstract advocacy unlikely to produce concrete action with actual advocacy, which is likely to produce concrete action.

Brandenburg is the Supreme Court’s most speech-protective formulation of an incitement test. A conviction for incitement under Brandenburg is constitutional only if several requirements are met: imminent harm, a likelihood of producing illegal action, and an intent to cause imminent illegality. None of the earlier tests had contained an intent requirement. Also, none ever had so clearly stated a requirement for a likelihood of imminent harm. Brandenburg does not answer, however, how imminence and likelihood are to be appraised.

Brandenburgh Applications[edit | edit source]

In NAACP v. Claiborne Hardware Co., 458 U.S. 886 (1982),[edit | edit source]

the Court overturned a judgment against the NAACP for a boycott of white-owned businesses that it alleged engaged in racial discrimination. In part, the trial court had based the liability of the NAACP for damages from the boycott on a speech by an NAACP official that included the statement, “If we catch any of you going in any of them racist stores, we’re gonna break your damn neck.”

The Court explained, “In the passionate atmosphere in which the speeches were delivered, they might have been understood as inviting an unlawful form of discipline or, at least, intending to create a fear of violence whether or not improper discipline was specifically intended. . . . This Court has made clear, however, that mere advocacy of the use of force or violence does not remove speech from the protection of the First Amendment. .

Holder v. Humanitarian Law Project[edit | edit source]

Facts: The Humanitarian Law Project (HLP), five other organizations, and two individuals (collectively Plaintiffs) filed suit in district court against U.S. Attorney General Eric Holder, Jr., and others (collectively Defendants) challenging the constitutionality of a federal law that prohibited the giving of “material support or resources” to certain foreign organizations designated by the Secretary of State as engaging in terrorist activities. 18 U.S.C. § 2339B(a)(1). Specifically, Plaintiffs sought to provide training, education, and other resources to two designated terrorist groups: the Kurdistan Workers’ Party, also referred to as the Partiya Karkeran Kurdistan (PKK) and the Libertarian Tigers of Tamil Eelam (LTTE).

Rules:

  • A material-support statute isn’t unconstitutionally vague.
  • A federal law barring material support to certain foreign organizations doesn’t infringe upon free speech or freedom of association.

Reasoning:

  • The ninth circuit failed to apply the rule that a person who engages in illegal conduct may not challenge the vagueness of the law as applied to others’ conduct. The statutory definitions of material support and related terms provide a person of ordinary intelligence with fair notice of what’s prohibited. Although the statute was not clear in every application, it is clear in their application to the proposed conduct. Likewise, the advocacy entirely independent of a foreign terrorist organization wouldn’t be unlawful.
  • The government’s interest in combatting terrorism is an urgent objective of the highest order. Therefore, congress was justified in concluding that material support to promote humanitarian or peaceful ends may be diverted by a terrorist group to illegal ends.

Fighting Words[edit | edit source]

The proceeding section focuses on when speech can be punished because it advocates illegal acts or the throwing of government. When may speech be punished because of the risk that it may provoke an audience into using illegal force against the speaker?

Chaplinsky v. New Hampshire[edit | edit source]

Facts: Appelant is a member of the Jahova’s Witness and was convicted in the municipal  court for violation of public law of NH saying “No person shall address any offensive, derisive, or annoying word to any other person who is lawfully in any street or public space” The complaint charged the appellant with “force and arms in a certain public place in Rochester on the public sidewalk on the easterly side of Wakefield street, did unlawfully repeat the words, addressed to the complainant, “you are a goddamned racketeer and a fascist”

Rules: “Fighting words” that incite others to violence are not protected by the First Amendment from governmental regulation

Reasoning: Even under the broadest meaning of the first amendment, the freedom of speech cannot be absolute. Punishment of certain narrow categories of speech has never been questioned under the Constitution. These categories included lewd, and obscene, profane, and libelous speech, as well as fighting words. Their very utterance, inflict injury or tend to incite an immediate breach of peace. This type of speech has very little social value and thus make no contribution to the marketplace of ideas protected by the first amendment.

Chaplinsky appears to recognize two situations where speech constitutes fighting words:

  • Where it is likely to cause a violent response against the speaker and
    1. As to the former, the danger that the listener will be provoked to fight, the issue is whether the appropriate response is to punish the speaker or rather to punish the person who actually resorts to violence.
  • where it is an insult likely to inflict immediate emotional harm.
    1. As to the latter, speech that inflicts an emotional injury, the question—which is key in the discussion of hate speech considered below—is whether speech should be punished because it is upsetting or deeply offensive to an audience.

Each aspect raises questions about whether such speech should be outside the protection of the First Amendment.

Fighting Words law invalidated as vague and overbroad[edit | edit source]

Gooding v. Wilson[edit | edit source]

Facts: The defendant was convicted in Georgia on two counts of using opprobrious words and abusive language to insult two Georgian cops.

Rules: While the First Amendment does generally not protect “fighting words”, a state statute prohibiting them may still be unconstitutional if it is over-inclusive.

Reasoning: The statute can only be upheld as constitutional if it prohibits words that fall under a narrow category of speech no protected by the 1st and 14th Amendments. Under Chaplinsky, states may punish the use of words that is blatantly insulting or meant to incite conflict. However, even in such a limited class, states must still be careful to narrowly draw their statutes so that they do not infringe upon protected speech. No state statute that purportedly criminalizes fighting words have been upheld because of the tendency to be overbroad. in criminalizing “opprobrious” and “abusive” speech, is over-inclusive because it encompasses more speech than just “fighting words.”

Narrow Fighting Word laws as Content-Based restriction[edit | edit source]

R.A.V v City of St Paul MN[edit | edit source]

Facts: The defendant, a juvenile, and several teens burned a wooden cross on the lawn owned by a clack family. They were arrested for violating the St Paul Bias Motivated Crime ordinance. The ordinance prohibited the placement of hateful symbols, which one knows or has reasonable grounds to know arouse anger, alarm or resentment in others on the basis of race, color, creed, religion, or gender.

Rules: Under the First Amendment, states may not regulate categories of unprotected speech, such as “fighting words,” on the basis of content.

Reasoning: A statute that regulates the content of speech on its face will only survive a constitutional challenge if it necessary to serve a compelling state interest. This means the statute will be struck down if there is a content-neutral alternative that satisfies the state’s objective.

The government is generally barred from regulating ideas expressed by speech or conduct. There are a number of traditional exceptions to this rule that allow the government to regulate speech with only ‘slight social value” such as fighting words. Nevertheless, the government may not use these exceptions to create content-based regulations. In this case, the Court is bound by Minnesota interpretation of the statute that regulates on fighting words within the meaning of Chaplinsky. Chaplinsky defines “fighting words” as “conduct that itself inflicts injury or tends to incite immediate violence.” Such language is not entirely without value but is not essential to the exposition of ideas. This implies that a state may not be permitted to regulate fighting words in all contexts. The constitutionality of such regulation ultimately depends on various elements of content expressed in speech and the secondary effect stemming from that content. The ordinance is unconstitutional despite its narrow construction. The statute specifically applies to fighting words that provoke violence “on the basis of race, color, creed, religion, or gender.” Under these terms, fighting words are permissible as long as they do not address one of the disfavored topics.

R.A.V can be appraised on many levels.[edit | edit source]

1. It can be analyzed in terms of what it means for the fighting words doctrine. Here, fighting words laws will be upheld only if it does not draw content-based distinctions among types of speech, such as by prohibiting fighting words based on race, but no based on political affiliation. The problem, is that it will be very difficult for legislation to meet this requirement without being overbroad and vague.

2. It can analyzed in terms of the Court’s holding that there is a strong presumption against content-based discrimination within categories of unprotected speech. This was the most divisive issue for the justices. On one hand, Scalia makes a powerful argument that the government should not be able to prohibit only obscenity or fighting words, but on the other, the concurring justices make a persuasive point that inevitably, in regulating categories of unprotected speech, the government will not forbid all such speech but draw lines. Such lines are vulnerable.

3. There is the question of whether the case should have been found to meet the exceptions that Scalia recognized where content-based distinctions within categories of unprotected speech should be allow. One instance is where the distinction advances the reason why the category is unprotected.

Hostile Audience[edit | edit source]

In some cases, especially in the 40s and 50s, the Supreme Court applied the clear and present danger test in dealing with the issue of when the government may punish individuals for speech that provokes a hostile audience. For example, in Terminiello v Chicago, the Court overturned a conviction for disturbing the peace because it was not shown that the speech posed a clear and present danger of lawlessness.

The court overturned the conviction and found that the jury instruction was not sufficiently protective of speech. The Court declared that a function of free speech under our system is invite dispute.

  • Cantwell v Connecticut – The Court overturned a conviction for disturbing the peace because of the absence of proof of a clear and present danger. Cantwell was JW and was prosecuted for playing music on a street corner that attacked the Roman Catholic religion. The Court found that there was no such clear and present menace to public peace and order.
  • In Feiner a guy was convicted for speech that he gave that sharply criticized the president and a local official for their inadequate records on civil rights. he Court quoted Cantwell to state that the government may prevent or punish speech that poses a clear and present danger. The Court concluded: “It is one thing to say that the police cannot be used as an instrument for the suppression of unpopular views .

The problem with the clear and present danger test in this context is that it allows an audience reaction, if hostile enough, to be a basis for suppressing a speaker. A speaker who is acting completely lawfully can be silenced because of illegal behavior—threats of violence and use of force—by members of the audience

  • Cox v Louisiana – Some members of the audience found the speech inflammatory and the speaker was arrested a day after the demonstration. The Court overturned the conviction and again emphasized the ability of the police to control the crowd. The Court stated that “it is virtually undisputed, however, that the students themselves were not violent and threatened no violence. The fear of violence seems to have been based on the reactions of the groups of white citizens looking from across the street
  • Gregory v City of Chicago – The Court unanimously overturned convictions for disturbing the peace for a group of demonstrators who had been arrested when an angry group threatened them. The demonstrators were marching to the mayor’s house when some members of the opposing group reacted angrily and threw rocks at them.

Problem of Racist Speech[edit | edit source]

Over the past few decades, there has been an important debate among scholars as to whether and when the government may punish racist speech. Those who favor the restrictions emphasize how hate speech undermines the constitutional value of equality. It is also argued that hate speech is form of verbal assault that should be punishable by law. But those who oppose such hate speech restrictions maintain that it is wrong to stop speech because it is distasteful and offensive. It is also argued that it is impossible to formulate a definition of racist speech that is not constitutionally vague and overbroad. It may also be used against minorities.

Virginia v. Black[edit | edit source]

Facts: In 1998, Barry Black (defendant) led a Ku Klux Klan rally in Virginia. At the close of the rally, the participants burned a cross. Black was charged with violating a Virginia statute that made it illegal to burn a cross if the burning was done with intent to intimidate someone. The statute also stated that the burning of a cross in itself is prima facie evidence of intent to intimidate. At Black's trial, the court instructed the jury that the burning of the cross itself was sufficient evidence from which to infer the required intent to intimidate.

Rules: A statute is unconstitutional if it both bans cross burning done with the intent to intimidate and states that the act of burning a cross is itself is a prima facie evidence of the intent to intimidate.

Holding: No, but the provision distinguishing cross burning as a prima facie evidence of intent to intimidate is unconstitutional

Reasoning: The first amendment protects not just speech, but also expressive conduct. Thus, cross burning perpetrated with the intent to convey a political or ideological message is expressive conduct. Nevertheless, there are some limits; such as constitutional limits on true threats.

True threats[edit | edit source]

This encompass those statements where the speaker means to communicate a serious expression of an intent to commit an act of unlawful violence to a particular individual or group of individuals.

According to the Court, intimidation is a type of true threat; therefore, cross burning perpetrated as an intimidation tactic may be prohibited. The prima facie requirement fails to distinguish between cross burning as an intimidating hate crime or a political speech.

the hallmark of the protection of free speech is to allow “free trade in ideas”—even ideas that the overwhelming majority of people might find distasteful or discomforting. Thus, the First Amendment “ordinarily” denies a State “the power to prohibit dissemination of social, economic and political doctrine which a vast majority of its citizens believes to be false and fraught with evil consequence.”

The fact that cross burnings are a symbolic expression does not resolve the constitutional question. In RAV, the court held it unconstitutional for a local ordinance banning certain symbolic conduct with the intent to arouse anger or violence because it was too vague and overbroad. The Court specifically stated that some types of content discrimination did not violate the first amendment, Indeed, we noted that it would be constitutional to ban only a particular type of threat; speech threatening the president.

Porn[edit | edit source]

Osborne v. Ohio[edit | edit source]

Facts: the state of Ohio criminalized the possession of child porn in order to prevent child exploitation. The relevant statute defined child porn as material that showed a child in a state of nudity and was possessed by a person who was not the child’s parent or guardian or who did not have permission from the child’s parent or guardian to possess the material for proper purpose. Osborne was convicted of the statute after being found in possession of explicit pictures of a child in his house.

Rules: The government may criminalize the private possession of child pornography.

Reasoning: In the case of child pornography, the government has an interest in protecting children from exploitation or from being further victimized through the resharing of explicit images. The government’s interest in protecting children is strong enough to allow the criminalization of the private possession of child pornography. The statute is well written to criminalize only the material that is harmful to children, providing exceptions for material that has a proper purpose. Therefore, the statute is not unconstitutionally overbroad.

Profanity and Indecent Speech[edit | edit source]

Cohen v. California[edit | edit source]

Facts: The LA Municipal Court convicted Rob Cohen for violating the state penal code prohibiting “maliciously and willfully disturbing the peace or quiet of any neighborhood or person by offensive conduct.” He was convicted after wearing a jacket bearing the words “Fuck the draft.”

Rules: Absent a particularized and compelling purpose, a state may not criminalize a public display of a single four-letter expletive without violating the First and Fourteenth Amendments.

Reasoning: The Conviction rested solely on the fact that women and children can freely see the message. Although the protection of women and children from such vulgarity was such a big public interest, protection of sensitive people alone is not enough to outweigh free speech. Offended viewers can simply avert their eyes. There are two exceptions for First Amendment protection, obscenity and fighting words, and fuck on a jacket did not belong to either one. The state can’t justify the conviction based on the state’s duty to protect public morals.

additionally, we cannot overlook the fact, because it is well illustrated by the episode involved here, that much linguistic expression serves a dual communicative function: it conveys not only ideas capable of relatively precise, detached explication, but otherwise inexpressible emotions as well. In fact, words are often chosen as much for their emotive as their cognitive force. We cannot sanction the view that the Constitution, while solicitous of the cognitive content of individual speech has little or no regard for that emotive function which practically speaking, may often be the more important element of the overall message sought to be communicated.

Finally, and in the same vein, we cannot indulge the facile assumption that one can forbid particular words without also running a substantial risk of suppressing ideas in the process. Indeed, governments might soon seize upon the censorship of particular words as a convenient guise for banning the expression of unpopular views. We have been able, as noted above, to discern little social benefit that might result from running the risk of opening the door to such grave results.

The O’Brien Test[edit | edit source]

Finding that Conduct communicates does not mean that it is immune from government regulation. The question then arises as to whether the government has sufficient justification for regulating the conduct.

United States v  O’Brien[edit | edit source]

Facts: On March 31, 1966, O’Brien and three others burned their Selective Service registration certificates on the steps of the Boston Courthouse. A sizeable crowd, including several agents of the FBI witnessed the event. For this act, O’Brien was tried and convicted and sentenced by the US District Court for District of Massachusetts.

Rules: Congress can Constitutionally prohibit the destruction or mutilation of draft cards.

If Conduct contains both speech and nonspeech elements, an important or substantial government interest in regulating the nonspeech element may justify incidental limitations on the protected speech if

  1. The regulation is within the Constitutional power of the government
  2. The Regulation furthers an important or substantial government interest
  3. The governmental interest is unrelated to the suppression of free expression
  4. The incidental restriction on alleged first amendment freedoms is no greater than is essential t the furtherance of the interest.

Reasoning:

We cannot accept the view that an apparently limitless variety of conduct can be labeled “speech” whenever the person engaging in the conduct intends thereby to express an idea. However, even on the assumption that the alleged communicative element in O’Brien’s conduct is sufficient to bring into play the First Amendment, it does not necessarily follow that the destruction of a registration certificate is constitutionally protected activity

The first amendment protects both speech and expressive conduct. The law prohibits destruction of draft cards dealt exclusively with conduct. The conduct at issue is not entirely expressive, because a person could burn a draft card with no intent to express an idea. Burning draft cards are no different from burning other government issued documents like driver’s licenses or tax forms. Additionally, the law targeted both public and private conduct. Because the law didn’t target conduct carried out for the sole purpose of expression, the statute was no facially unconstitutional. O’Brien engaged in the conduct at issue in order to express an idea, thus combining the speech and nonspeech elements. If Conduct contains both speech and nonspeech elements, an important or substantial government interest in regulating the nonspeech element may justify incidental limitations on the protected speech elements. The government’s interest was sufficiently important to justify the regulation, despite any incidental limits on free speech.

  1. The draft is within the Congress’s constitutional war powers
  2. Interests was in classifying citizens for the draft and prevent destruction of draft cards
  3. Purpose for the statute was not to suppress expression
  4. Legislation narrowly constructed to preserve that interest.

Flag Desecration[edit | edit source]

A major area where the Supreme Court has applied the O’Brien test is with regard to flag burning and flag desecration laws. After initial cases that protect flag desecration on narrow grounds, but without resolving the issue, the Court in 1989 and again in 1900 made it clear that flag burning is a constitutionally protected speech.

Texas v. Johnson[edit | edit source]

Facts: Gregory Lee Johnson (defendant) burned an American flag during a political demonstration at the 1984 Republican National Convention in Dallas. The State of Texas (plaintiff) charged Johnson with desecration of a venerated object in violation of a state statute. Johnson was convicted, sentenced to one year in prison, and fined $2,000. Johnson appealed his conviction.

Rules: Flag burning is expressive conduct protected by the first amendment and a statute that criminalizes the burning of an American flag as a means of political protest violates the first Amendment.

Reasoning: The First Amendment protects flag burning because it contains a sufficient level of communication. By burning the flag, Johnson tried to convey an idea which any onlookers would understand the meaning of. In First Amendment cases, the states have more leeway in regulating expressive conduct than written or expressive words. Applying a similar test to O’Brien, the state must show that the law was for an important government interest and that it was unrelated to suppressing expression. Texas said that its government interest was to protect the speech; which was not implicated in Johnson’s case. Second, Texas asserted that they wanted to keep the flag as a symbol of national unity; but this is related to a political message and did not qualify for the lenient standard. Because the regulation of speech was content-based, the Court submitted the case under the most exacting constitutional scrutiny; under that scrutiny, it was found that preserving the flag as a symbol did not justify the punishment.

Johnson was not, we add, prosecuted for the expression of just any idea; he was prosecuted for his expression of dissatisfaction with the policies of this country, expression situated at the core of our First Amendment values. If he had burned the flag as a means of disposing of it because it was dirty or torn, he would not have been convicted of flag desecration under this Texas law. The Texas law is thus not aimed at protecting the physical integrity of the flag in all circumstances, but is designed instead to protect it only against impairments that would cause serious offense to others. Texas law thus depended on the likely communicative impact of his expressive conduct.

Distinction Between Content-Based and Content-Neutral Laws[edit | edit source]

The Supreme Court frequently has declared that the very core of the First Amendment is that government cannot regulate speech based on its content. The Court stated, above all else, the First Amendment means that government has no power to restrict expression because of its message, its ideals, its subject matter or its content.

In Turner Broadcasting System v FCC, the Court explained the content-based restrictions generally must meet strict scrutiny, while content neutral laws only need to meet intermediate scrutiny.

Reed v. Town of Gilbert[edit | edit source]

Facts: Reed ran a church whose place of worship varied from week to week. Thus, to notify his congregation, he posted signs outside week to week, often removing them after church service has concluded. The town of Gilbert found this illegal, nothing that its sign code prevents the display of outdoor signs unless the signs fall within an exempt category. These categories permit signage, but with restrictions. The restriction prevented signs like these, i.e. directional signs, from being kept posted for more than 12 hours.

Rules: Categorizing signs based on their purpose is content-based speech regulation and is subject to strict-scrutiny review.

Reasoning: The first amendment, applicable to the States through the Fourteenth Amendment, prohibits restrictions on speech based on its message. If a law is based on content, the government must show that the law is tailored narrowly to serve a compelling state interest. A law is content based if it restricts the topic, idea, or type of message of the speech. A neutral justification can’t save a law that’s content based. The sign code classifies categories of exemptions based on the subject of the speech and is therefore content based.

Why is there so much concern about content neutrality? Obviously, the fear is that the government will target particular messages and attempt to control thoughts on a topic by regulating speech. The government could try to control dissent and advance its own interests by stopping speech that expresses criticism of government policy, while allowing praise. A subject-matter restriction on speech can accomplish the same goal.

How is it determined whether a law is content based?[edit | edit source]

The requirement that the government must be content-neutral in its regulation of speech means that the government must be both viewpoint neutral and subject matter neutral. In other words, a law will be found to be content based if it is either a viewpoint or subject matter restriction. Viewpoint-neutral means that the government cannot regulate speech based on the ideology of the message.

Matal v Tam[edit | edit source]

Facts: Tam had a band and wanted the name “The Slants” as a band name to reclaim the derogatory term since he and his bandmates were Asians. The Lanham act prohibited the registration of trademarks that disparages any person, whether living or dead.

Rules: The law’s prohibition on disparaging trademarks violates free speech because it discriminates based on viewpoint, singling out marks that offends persons and groups.

Reasoning: In interpreting the statute, the prohibition on registering trademarks that disparages persons covers racial and ethnic groups; thus applying to Tam’s trademark. Generally, a government regulation that favors one viewpoint and restrains another is unconstitutional.

The Court considered 3 categories meriting Reduced or No Scrutiny.

  • Are trademarks government speech? – a narrow category of expression that requires extensive government involvement and is exempt from first amendment scrutiny. If government itself speaks, then it doesn’t have to be viewpoint neutral.
    1. But trademarks aren’t government speech. The government doesn’t make, edit, or regulate trademarks, just approves them.
    2. Trademarks are private speech subject to scrutiny
  • Is trademark registration a government subsidy? – These tolerate viewpoint discrimination. Free speech doesn’t compel the government to promote things that it doesn’t want to.
    1. But trademarks aren’t government subsidy.
  • Are trademarks commercial speech? – these are subject to relax first amendment scrutiny. Restrictions here must be narrowly tailored to serve a substantial government interest.
    1. Even if it were commercial speech, the disparagement clause could not withstand relaxed scrutiny. The clause was intended to encourage the free flow of commerce and to shield underrepresented groups from exposure to demeaning advertising. But demeaning speech is protected by the first amendment, the clause wasn’t narrowly drawn to serve the government’s purpose.

Two years later, in Iancu v Brunetti, the court struck down another provision of the Lanham Act, which prohibited registration of trademarks which are scandalous or immoral. A clothing manufacturer wished to register as a trademark “FUCT.” The court held that it was viewpoint based because of the act’s criteria for “immoral or scandalous” trademark. The meanings of “immoral” and “scandalous” are not mysterious, but resort to some dictionaries still helps to lay bare the problem. When is expressive material immoral? According a standard definition, when it is inconsistent with rectitude, purity, or good morals. So the Lanham Act permits registrations of marks that champion society’s sense of morality but not ones that denigrate from those concepts.

Vagueness and Overbreadth[edit | edit source]

Laws that regulate speech can be challenged as facially unconstitutional on the grounds that they are unduly vague and overbroad. A successful challenge usually means that the laws is entirely invalidated, as opposed to being declared unconstitutional as to certain application.

Vagueness[edit | edit source]

A law is unconstitutionally vague if a reasonable person cannot tell what speech is prohibited and what is permitted. It is important to emphasize that unduly vague laws violate the due process whether or not speech is regulated. In part, vagueness doctrine is about fairness, it is unjust to punish a person without providing clear notice as to what conduct was prohibited. Vague laws also risk selective prosecution; under vague statutes and ordinances, the government can choose who to prosecute based on their views or politics.

If a law fails to provide such minimal guidelines, a criminal statute may permit a standard sweep that allows policemen, prosecutors, and juries to pursue their own personal predilections. Although a law regulating conduct ca be challenged under the due process vagueness doctrine, courts are particularly troubled about vague laws restricting speech out of concern that the will chill constitutionally protected speech.

Coates v. Cincinnati[edit | edit source]

Facts: Coates and co were convicted of violating an ordinance of the city of Cincinnati that made it a criminal offense for three or more people to assemble on a city sidewalk and there conduct themselves in a manner annoying to persons passing by. Coates appealed his conviction on the ground that the city ordinance was unconstitutionally vague. Coates’ conviction was upheld by the state supreme court and Coates petitioned the United States Supreme Court for Review.

Rules: A city’s criminal ordinance can be facially unconstitutional if it’s vague or overbroad.

Reasoning: Although a city can constitutionally adopt ordinances to prohibit certain antisocial conduct, the city can’t constitutionally adopt ordinances that are vague or overbroad. An ordinance is unconstitutionally vague if it is unclear to an average person of common intelligence what conduct is actually prohibited. If a statute is vague, then it violates the fourteenth amendment’s due process protections if it subjects a person to criminal penalties without providing adequate prior notice.

An ordinance can become unconstitutionally overbroad if it penalizes constitutionally protected conduct. The First Amendment protects freedom of assembly and association, which the ordinance penalizes. There could be discrimination in enforcement of an overbroad law because enforcers can pick and choose what the broad law defines.

The ordinance failed to define what annoying was and therefore failed to give adequate notice to Coates and co.

Overbreadth[edit | edit source]

A law is unconstitutionally overbroad if it regulates substantially more speech than the Constitution allows to be regulated, and as a person to whom the law constitutionally can be applied can argue that it would be unconstitutional as applied to others. A person’s speech Is unprotected by the First Amendment and who could constitutionally be punished under a narrow statute may argue that the law is unconstitutional because of how it might be applied to third parties not ever the court.

Schad v. Borough of Mount Ephraim[edit | edit source]

Facts: Schad opened an adult bookstore in the Borough of Mount Ephraim. He installed a coin operated device which allowed customers to watch a stripper. The live entertainment service was deemed to be a violation of the commercial zoning ordinance. The ordinance contains an exhaustive list of permissible commercial uses and did not include live entertainment.

Rules: A law is impermissibly overbroad if it excludes a broad category of protected expression without justification.

Reasoning: The first amendment, as incorporated into the states through the fourteenth amendment, requires states to prove a sufficient justification for laws that exclude a broad category of protected expression. When a law infringes on a protected liberty, the laws must be narrowly drawn to further a sufficiently substantial government interest.

Here, the Borough argues live entertainment will prevent their desire to create a commercial zone, but the law lists a range of commercial uses that do more than create a commercial zone to meet the immediate needs of residents. They also argue that they banned live entertainment to prevent the harm associated with it, the law shows no evidence that live entertainment is more harmful than others. The Borough has also failed to prove its interests can’t be met by restrictions that are less intrusive on protected forms of expressions.

Although the power of a local government to zone and control land is broad, this power isn’t unchallengeable and must be exercised within constitutional limits.

In subsequent cases, the Court made it clear that the requirement for substantial overbreadth applies in all cases, whether the law regulates conduct that communicates pure speech. A statute may be invalidated on its face, only if the overbreadth is substantial.

The concept of substantial overbreadth is not readily reduced to an exact definition. It is clear however, that the mere fact that one can conceive of some impermissible applications of a statute is not sufficient to render it susceptible to an overbreadth challenge. In short, there must be a realistic danger that the statute itself will significantly compromise recognized First Amendment protections of parties not before the court to make it facially challenged on overbreadth grounds.

It appears, then, that substantial overbreadth might be demonstrated by showing a significant number of situations where a law could be applied to prohibit constitutionally protected speech. In contrast, if the Court believes that the law will apply to relatively few situations where speech is constitutionally protected, it will not be declared overbroad.

The second major aspect of the overbreadth doctrine is that a person to whom the law constitutionally may be applied can argue that it would be unconstitutional as applied to others. The usual rule of standing is that a person to whom a statute may constitutionally be applied will not be heard to challenge that statute on the ground that it may conceivably be applied unconstitutionally to others.

Relationship between vagueness and overbreadth[edit | edit source]

The concepts of vagueness and overbreadth are closely related; laws often are challenged under both of these doctrines simultaneously. These concepts are best understood as overlapping and not identical.

Board of Airport Commissioners v. Jews for Jesus, Inc.[edit | edit source]

Facts: The board of airport commissioners made a resolution saying that all first amendment activities by any individual or entity are banned in the Central Terminal Area and violators are subject to legal action. A member of Jews for Jesus was handing out religious pamphlets and was asked to leave or face legal actions based on the resolution.

Rules: Legislation banning all First Amendment speech is unconstitutional unless it’s subject to a limiting construction.

Reasoning: The First Amendment overbreadth doctrine states that a person whose speech may be prohibited can challenge a statute on its face because it also threatens others not before the court. But it can only be invalidated on its face if the overbreadth is substantial and there’s realistic danger that the statute will harm other’s First Amendment rights. An overbroad statute may still be valid if courts can narrow its construction. If state courts haven’t defined the statute’s constructions, the Supreme Court can only do so upon certification from the state or abstention. It was overbroad, and it could not be limited because California had no procedure to define resolution’s construction and abstention would make it unfeasible to provide a limiting construction in a limited time frame.

The resolution was so broad that any limiting construction would require a lot of case-by-case adjudication and if limited to prevent speech not having to do with the airport, it would still be overly vague.

Defamation[edit | edit source]

Under state statute Public Official/Person Private Person
State of Mind Defendant is liable if they can say NYT v Sullivan Gertz v R. Welch
Liability P and P D’s
Knowledge “ I knew it was false, but said it anyway” Okay for the State to Punish OK OK
Recklessness “I did not know if it was true or false but didn’t care, so I said it anyway” Okay for the State to Punish OK OK
Negligence “I thought it was true, but I didn’t do much to check and said it” State may not punish OK NO
Strict Liability “I thought it was true, and I did all I could to check, so I said it” State may not punish NO NO

Public/private figures[edit | edit source]

  • Dun and Bradstreet case – private figure, private issue

Generally,[edit | edit source]

  • If dealing with public official or figure – apply NY Times v. Sullivan
  • If dealing with private figure/public issue – apply Gertz
  • If dealing with private figure/private issue – common law applies

Reputation, Privacy, Publicity, and the First Amendment[edit | edit source]

Many tort claims seek to impose liability for speech. Similarly, the false light tort is liability for speech that creates a false impression about a person and his or her activities. Speech can be the basis for a claim for the intention infliction of emotional distress. Also, speech that discloses private information or exploits the commercial likeness of another may be the basis for a tort of invasion of privacy or for violating the right to publicity.

New York Times v. Sullivan[edit | edit source]

Facts: Sullivan was a public official of Alabama and sued New York Times and several others on defamation grounds. It is undisputed in the case that the statements in the Times were all either exaggerations or completely false, thus being libelous per se.

Rules: If the plaintiff is a public official or is running for office, they can recover damages for defamation only by proving clear and convincing evidence that the defamatory statements were a lie and that there was malice in the speaker.

Reasoning: The Constitutional safeguard was fashioned to assure unfettered interchange of ideas for to bring political and social changes desired by the people. If alone, neither factual error nor defamatory content suffices to remove the constitutional shield from criticisms of official conduct, then the combination of the two would be no less inadequate. Compelling critics to guarantee the truth in all their assertions is self-censorship. The Constitution require a federal rule that prohibits a public official from recovering damages for a defamatory lie relating to their conduct unless they prove that the statement was made with “actual” malice.

Requirements of this case[edit | edit source]

  1. The plaintiff must be a public official or is running to be one.
  2. The plaintiff must prove his or her case with clear and convincing evidence
  3. The plaintiff must prove falsity of the statement
  4. The plaintiff must prove actual malice ( knowledge of falsity but disregarded it)

Public Figures as Plaintiffs[edit | edit source]

The Supreme Court has held that the same rules apply in defamation suits brought by public figures. In Curtis Publishing v Butts and Associated Press v Walker, both cases had plaintiffs who were not public officials but were very prominent in their community.

There was not a majority in either case, but a plurality opinion did say that although the plaintiffs were not officials, the public interest in the circulation of the materials and the publisher’s interest in circulating them, is not less than that in NYT v Sullivan.

Gertz v. Robert Welch, Inc.[edit | edit source]

Facts: Gertz was an attorney that sued a cop for wrongful death on behalf of the victim’s family. American Opinion is a magazine that made statements about a grand Communist conspiracy against the Government and noted that Gertz was one of these Communists trying to ruin the reputation of law enforcers. In their magazine, they painted Gertz as an architect of anarchy against the officer whom he supposedly framed.

Rules: The First Amendment does not allow newspaper or broadcasters to assert defamatory falsehoods against private individuals in the same capacity as it would with public people.

Reasoning: Although there is no place for falsehoods in the marketplace of ideas, they are an inevitability in the never-ending debate of society. The first amendment recognizes that this debate allows the defamatory information about public individuals, so long as no malice exists, as preventing such criticism would involve self-censorship. However, a private individual should not be put under the same scrutiny and deserves more protection from defamatory falsehoods. Gertz was never a public individual, his notoriety were part of the case’s coverage and was not something he consented to; likewise, any influence he may have is only incidental. Private people have far less of a platform to rebut and protect themselves for defamatory falsehoods compared to public individuals.

In Dun & Bradstreet v Greenmoss Buliders[edit | edit source]

the Court said that the distinction must be drawn in suits against private figures between speech that involves matters of public concern and those that do not. Not all speech is of equal first amendment importance. It is speech on matters of public concern that are at the heart of first amendment protection. In contrast, speech on matters of purely private concern is of less important to the first amendment.

If public concern is defined in terms of the public’s actual interest, the media’s judgment to publish is likely strong, if not conclusive, evidence of people’s interest in the material. But if public concern is defined from a more objective viewpoint, then Courts are in the position of deciding what people in their enlightened best interest, should want to know.

How about lying?[edit | edit source]

  • Alvarez case re Stolen Valor Act
    • Is the statute constitutional?
      • Does it involve speech – yes
      • Does it infringe speech – yes
      • Is it overbroad or vague – not as far as the majority is concerned
      • Is it content-based – yes
      • Is it within a less-protected class – no
  • Key parts of the opinion:
    • “The Court has never endorsed the categorical rule the Government advances: that false statements receive no First Amendment protection”
    • Clarification re role of rule regarding knowing and reckless disregard
    • Distinction between perjury and other prohibitions
    • Concern re reach of the statute and with fear of slippery slope
    • Fails under strict scrutiny means-test prong

Intentional Infliction of Emotional Distress[edit | edit source]

Hustler Magazine v Falwell[edit | edit source]

Facts: Hustler Magazine made a parody about Minister Falwell that grossly depicted him as a motherfucker.

Rules: Public people may not recover for the tort of intentional infliction of emotional distress by reason of publications as the one at issue here without showing that the publication contains a false statement of fact made with “actual” malice.

Reasoning: Although the parody here was unpleasant and disgusting, it is no different from political editorial cartoons that depict public people in outrageous ways.

Snyder v Phelps[edit | edit source]

Facts:  The Snyders lost a son in Afghanistan and they were burying him. Phelps and his Westboro congregation picketed a couple of feet away from Snyder’s funeral, touting offensive words regarding homosexuals.

Rules: A church protesting a military funeral on public land in a peaceful manner is considered public speech protected by the First Amendment.

Reasoning: It was on public land and it was speech regarding public concern. Likewise, what inflicted the distress was the contents of the speech and not the manner of the protest itself.

Obscenity[edit | edit source]

Roth v United States[edit | edit source]

Issues: The issue is whether the federal obscenity act violated the First Amendment protection of free speech

Rules: Obscenity is a low value form of speech that is not protected by the first amendment

Holding: No

Reasoning: Historically, the first amendment was used to protect speech that has some inherent social value; typically ones that may contribute to social changes in the marketplace of ideas. Obscenity is seen as having little value in this regard and is not protected by the first amendment.

Paris Adult Theater v Slaton[edit | edit source]

Facts: Owners and operators of a porn movie theater argued that the civil cases against them were unconstitutional because their theaters show protected content under the first amendment. They argue that because only consenting adults come into their theater then it would not be obscene.

Rules: In Roth v United States, the Supreme Court held that obscenity is not a protected form of speech under the First Amendment.

Reasoning: Besides the above reasoning; the States also have a vested interest in keeping their community safe, and such safety and enrichment may include the regulation of obscene materials. Thus, a state may do whatever it sees fit in regulating these obscene materials.

Obscenity Standards[edit | edit source]

Miller v California[edit | edit source]

Facts: Miller sent, to the mail, a bunch of illustrated adult books and was penalized by the state of California under its criminal obscenity laws for distributing obscene materials to people who have in no way of consenting to receiving such materials.

Rules: A State may regulate obscene materials without infringing on the First Amendment as applicable to the States through the Fourteenth Amendment

Reasoning: Besides the above reasoning. The court also used this case to set certain limits on Constitutional protection of obscene materials. The basic factual guidelines are

  1. Whether the average person, applying contemporary community standards, would find the work as appealing to prurient interest.
  2. Whether the work depicts or describes, in the patently offensive way, sexual conduct specifically defined by the applicable state law
  3. Whether the work, taken as a whole, lack serious literary, artistic, political, or scientific value.

Under the holdings announced today, no one will be subject to prosecution for the sale or exposure of obscene materials unless these materials depict or describe patently offensive “hard core” sexual conduct specifically defined by the regulating state law.

Child Pornography[edit | edit source]

New York v Ferber[edit | edit source]

Issues: At issue in this case is the constitutionality of a New York criminal statute which prohibits persons from knowingly promoting sexual performances by children under the age of 16 by distributing material which depicts such performances

Rules: The Miller standard is incompatible with laws against child pornography, but there is a huge, vested interest in the state to suppress this sort of obscenity that great surpasses any Constitutional limits.

Holding: It is Constitutional

Reasoning: In order for a State actor to create statutes that violate Constitutional provisions, that State actor must have an interest that will justify its enactment. Here, the protection of minors from sexual abuse is that justification. Although, this test is unnecessary as Child pornography is already seen as obscene and therefore unprotected by the first amendment.

However, beyond safeguarding the rights of minors, the Court also had other reasons. Such as, these materials are permanent records of child exploitation and abuse. Third, the sale of this content is unanimously seen as illegal across the nation. Fourth, it is unlikely for this content to have any value in literature, art, science, or education. Fifth, suppressing this content will not be inconsistent with past decisions.

The government cannot ban child pornography based on its condemnation of the material. Rather, the government’s interest is limited to protecting children from being used in the making of the material.

Protected but Low-value Sexual speech[edit | edit source]

The Supreme Court has indicated there is a category of sexual speech that does not meet the test for obscenity and thus is protected by the First Amendment, but is deemed to be speech of low value, and thus the government has latitude to regulate such expression. The Court never has defined the contours of this category, but it clearly involves sexually explicit material.

Zoning Ordinances[edit | edit source]

Young v American Mini Theaters[edit | edit source]

Facts: Zoning ordinances adopted by the city of Detroit differentiate between motion picture theaters which exhibit sexually explicit “adult” movies and those which do not

Issues: The principal question presented by this case is whether that statutory classification is unconstitutional because it is based on the content of communication protected by the First Amendment.

Rules: Since what is ultimately at stake is nothing more than a limitation on the place where adult films may be exhibited, even though the determination of whether a particular film fits that characterization turns on the nature of its content, we conclude that the city’s interest in the present and future character of its neighborhoods adequately supports its classification of motion pictures.

Holding: It is constitutional

Reasoning: The first amendment protects low value speech that may have some sort of artistic, literary, educational, or scientific value. Here, theater porn is considered to have some artistic value. Likewise, there is some wisdom in the state actor to differentiate between adult theaters and regular theaters. Additionally, the zoning laws do not deter the presentation and establishment of these theaters, just that they are not allowed to be within a same area; local ordinances do this regularly with other businesses regardless of content. Therefore, there is no infringement of Constitutional rights.

Government Techniques for Controlling Obscenity and Child Pornography[edit | edit source]

Stanley v Georgia[edit | edit source]

Facts: The dude was suspected for his book marking activities, Cops searched his houses, but they found not evidence relating to that, but they did find film reels of obscene material.

Rules: prosecuting the mere possession of obscene materials in one’s home is a violation of the first amendments

Reasoning: Although States do have a reason to regulate the distribution of obscene material, the ownership of these materials does not constitute a punishable violation, and even more so in one’s own home. Given the present state of knowledge, the State may no more prohibit mere possession of obscene matter on the ground that it may lead to antisocial conduct than it may prohibit possession of chemistry books on the ground that they may lead to the manufacture of homemade spirits.

Osborne v. Ohio[edit | edit source]

Facts: Ohio (plaintiff) criminalized the possession of child pornography in order to prevent the exploitation and revictimization of children. The relevant statute defined child pornography as material that showed a child in a state of nudity and was possessed by a person who was not the child’s parent or guardian and who did not have permission from the child’s parent or guardian to possess the material for a proper purpose.

Rules: the government can criminalize the private possession of child pornography

Reasoning: The government may criminalize the private possession of child pornography. The freedoms enshrined in the First Amendment may be limited if the government can show a strong enough interest. In the case of child pornography, the government has an interest in protecting children from exploitation or from being further victimized through the resharing of explicit images.

The Broadcast Media[edit | edit source]

FCC v Pacifica Foundation[edit | edit source]

Facts: A radio station played George Carlin’s seven dirty words and was reported to the FCC and was about to be sanctioned for it.

Rules: Under the First Amendment, the Federal Communications Commission may regulate a radio broadcast that is indecent but not obscene.

Reasoning: Such vulgar words have very little social value, but although it can have value given the right context, it is the fact that the radio station was very accessible that context would not matter. Many people are tuning in and out of the station, people in their own home who have a reasonable expectation of privacy, and impressionable kids may not know the context of the monologue, nor would they be aware of the prior warnings. Thus, the government is allowed such regulations.

The Internet[edit | edit source]

Reno v ACLU[edit | edit source]

Facts: The “indecent transmission” provision of the Communications Decency Act of 1996 (CDA) prohibited the knowing transmission of obscene or indecent messages via the internet to any recipient under the age of eighteen. The “patently offensive display” provision of the CDA prohibited the knowing, sending, or displaying of patently offensive messages in a manner that is available to a person under eighteen years of age.

Rules: Under the First Amendment, the government may not regulate the transmission and display of content on the internet unless it does so for a compelling purpose and uses means that are narrowly tailored to that purpose.

Reasoning: The internet was entitled to a more expansive first amendment freedom because websites do not intrude onto the viewers, but is sought after. Likewise, the internet is on limited frequencies, it is for everyone. The statute was also vague and overbroad

Schools[edit | edit source]

Tinker v. Des Moines Independent Community School District[edit | edit source]

Fact: A couple of kids at school wore black armbands to show their disdain for the Vietnam war draft. The school took away their armbands and punished the students.

Rules: In a public-school setting, prohibiting an expression of an opinion is unconstitutional unless there is a specific showing that engaging in the forbidden conduct would materially and substantially interfere with appropriate discipline in the operation of the school.

Reasoning: The Constitution asks us to take risks involving the freedoms of speech. The problem posed is not the simple regulation of skirts and clothes nor does it involve the aggressive or disruptive action or even group demonstration. It is directly affecting pure speech; passive expression of opinion unaccompanied by any disorder or disturbance.

The Court applied Tinker to the college context, in Papish v Board of Curators where it was held that a student could not be expelled for a political cartoon in a newspaper. In more recent years, the Court has been much less protective speech in school environments and much more deferential to school authorities.

Bethel School District No. 403 v. Fraser[edit | edit source]

Facts: Fraser, a high school senior who was suspend for three days by the school after he gave a lewd speech in front of the school during an assembly.

Rules: Students do not shed their first amendment rights when entering the school gates. Schools may properly punish students’ speech with suspension if they determine that speech to be lewd, offensive, or disruptive to the school’s basic educational mission.

Reasoning: Surely it is a highly appropriate function of public school education to prohibit the use of vulgar and offensive terms in public discourse. Nothing in the constitution prohibits the states from insisting that certain modes of expressions are inappropriate and subject to sanctions. The school must teach students beyond just books, this includes through moral values. The pervasive sexual innuendo in Fraser’s speech was plainly offensive to both teachers and students – indeed to any mature person. By glorifying male sexuality and its verbal content, the speech acutely insulting teenage girls. Speech could be seriously damaging to its less mature audience of 14 year olds.

Morse v. Frederick[edit | edit source]

Facts: Frederick, during a school sanctioned televised event, showed a 14-foot tall banner saying “Bong hits for Jesus” and was punished by the school.

Rules: Under the First Amendment, school officials may prohibit student speech that can reasonably be interpreted as promoting illegal drug use

Reasoning: Similar to the above reasoning. However, this has an additional caveat of how the defendant claims that the speech was nonsense and did not mean anything except to attract television cameras. However, it was plainly reasonable for the principal to interpret the message as advocating for drug use as kids know that bong hit is related to marijuana use.

When must the government make content-based choices?[edit | edit source]

For instance, if the government is choosing to subsidize speech, there is no way that it can avoid content considerations in deciding what to finance. The Court indicated that in such circumstances the government must be viewpoint-neutral but otherwise can consider content.

National Endowment for the Arts v Finley[edit | edit source]

Facts: The National Foundation on the Arts and Humanities Act (NFAHA) requires the Chairperson of the National Endowment for the Arts (NEA) to ensure that “artistic excellence and artistic merit are the criteria by which grant applications are judged, taking into consideration general standards of decency and respect for the diverse beliefs and values of the American public.” Finley and three others (plaintiffs) were performance artists who applied for NEA grants. Finley was informed that she had been denied funding, and brought suit against the NEA (defendant) in district court. Finley alleged that the NEA’s grant-awarding policy violated their First Amendment rights.

Rules: In making determinations regarding recipients of government subsidies, the government may consider content but must remain viewpoint-neutral.

Additional Rule: Facial invalidation “is, manifestly, strong medicine” that “has been employed by the Court sparingly and only as a last resort.” To prevail, respondents must demonstrate a substantial risk that application of the provision will lead to the suppression of speech.

Reasoning: In the competition for government funding, the government may choose to prioritize spending on one form of speech or activity over the other. This is not exclusion and it is also not impermissible because criminal penalty is not at stake. Any content-based considerations that may be taken into account in the grant-making process are a consequence of the nature of arts funding. The very assumption of the NEA is that grants will be awarded according to the artistic worth of competing applications; absolute neutrality is impossible. If a subsidy were manipulated to have a coercive effect, then judicial relief is an appropriate remedy. However, unless and until the NFAHA is applied in a manner that raises concern about the suppression of disfavored viewpoints, it is constitutional.

Government Speech[edit | edit source]

Pleasant Grove City, Utah v Summum[edit | edit source]

Facts: Summum (plaintiff), a religious organization headquartered in Salt Lake City, Utah, filed suit and a request for a preliminary injunction in federal court against the City of Pleasant Grove, Utah, (City) (defendant) after the City refused to erect a permanent stone monument that contained the Seven Aphorisms of Summum in a public park. The religious group claimed the City’s refusal violated Summum’s First Amendment rights.

Rules: although a park is a traditional public forum for speeches and other transitory expressive acts, the display of a permanent monument in a public park is not a form of expression to which forum analysis applies. Instead, the placement of a permanent monument in a public park is best viewed as a form of government speech and is therefore not subject to scrutiny under the Free Speech Clause.

Reasoning: Often, statutes in public parks are seen as Government speech. While government speech is not restricted by the Free Speech Clause, the Government does not have free hand to regulate private speech on government property. Just as government-commissioned and financed monuments speak for the government, so do privately financed and donated monuments that the government accepts and displays on government land. If government entities must maintain viewpoint neutrality in their selection of donated monuments, they must either “brace themselves for an influx of clutter” or face the pressure to remove longstanding and cherished monuments. To be sure, there are limited circumstances in which the forum doctrine might properly be applied to a permanent monument. Here, the City’s decision to reject Summum’s monument is best viewed as a form of government speech and is therefore not subject to scrutiny under the Free Speech Clause.

Walker v Texas Division, Sons of Confederate Veterans[edit | edit source]

Facts: Texas offers automobile owners a choice between ordinary and specialty license plates. Those who want the State to issue a particular specialty plate may propose a plate design, comprising a slogan, a graphic, or (most commonly) both. If the Texas Department of Motor Vehicles Board approves the design, the State will make it available for display on vehicles registered in Texas. The Texas Division of the Sons of Confederate Veterans proposed a specialty license plate design featuring a Confederate battle flag. The Board rejected the proposal.

Rules: Generally, the First Amendment cannot be used to challenge government speech.

Reasoning: Just like with how the Government cannot compel the SCV to express speech they do not agree with, the SCV cannot compel the same of the government. Thus, the Government is not required by the First Amendment permit the SCV’s license plates.

When government speaks, it is not barred by the Free Speech Clause from determining the content of what it says. That freedom in part reflects the fact that it is the democratic electoral process that first and foremost provides checks on government speech. The free speech clause instead helps produce informed opinion among members of the public, who then are able to influence the government, not the other way around.

Rust v Sullivan[edit | edit source]

Facts: In 1970, Congress enacted Title X of the Public Health Service Act (Act), which provides federal funding for family-planning services. The Act authorizes the p. 1296Secretary to “make grants to and enter into contracts with public or nonprofit private entities to assist in the establishment and operation of voluntary family planning projects which shall offer a broad range of acceptable and effective family planning methods and services.”

the Secretary of Health and Human Services, issued new regulations that attached three principal conditions on the grant of federal funds for Title X projects. Firstly, Title X projects could not provide counseling concerning the use of or provide referral for abortion as a method of family planning. Secondly, projects could not engage in activities that encouraged, promoted, or advocated abortion as a method of family planning. Thirdly, Title X projects were required to be organized so they were physically and financially separate from prohibited abortion activities.

Rules: A federal law may, as a condition of receiving federal funds, constitutionally restrict fund recipients from engaging in abortion-related activities.

Reasoning: Rust argues that the regulation prohibits “all discussion about abortion as a lawful option – including counselling, referral, and the provision of neutral and accurate information about ending a pregnancy – including counselling the clinic to provide more information that promotes continuing the pregnancy. There is no question that the regulations are Constitutional. The government’s asserted purpose for the regulations is to encourage family planning rather than provide prenatal care. Thus if certain activities are not funded, then it is because they are out of scope of the program. To hold that the government unconstitutionally discriminates on the basis of viewpoint when it chooses to fund a program dedicated to advancing certain permissible goals, would render numerous government programs constitutionally suspect. Rust is entirely free to pursue an abortion outside the receipt of the funds. Thus, the regulations do not violate her first amendment rights.

Legal Services Corp. v. Velazquez[edit | edit source]

Facts: In 1974, Congress enacted the Legal Services Corporation Act. The Act establishes the Legal Services Corporation (LSC) as a District of Columbia nonprofit corporation. LSC’s mission is to distribute funds appropriated by Congress to eligible local grantee organizations “for the purpose of providing financial support for legal assistance in noncriminal proceedings or matters to persons financially unable to afford legal assistance.”

Rules: Federal laws prohibiting lawyers from receiving federal funds from challenging existing laws violate the First Amendment.

Reasoning: Under Rust, the restriction were deemed necessary to enforce the limits of the Title X project, and thus, did not Constitute invidious discrimination against a particular viewpoint. However, this wide constitutional latitude for governmental speech does not always apply to instances in which the government provides subsidies to private speakers. This is particularly relevant to the LSC as it was designed to facilitate private speech rather than a government message. Additionally, the government created a subsidy to further a specific purpose.

At the same time, however, the government places a substantial limitation on this speech prohibiting LSC-funded lawyers from giving advice to clients regarding the Constitutional validity of Federal welfare statutes. By limiting this type of advice or communication, the government not only restricts speech in this arena, but also threatens to severely impair the judiciary.

The LSC funding restriction is unconstitutional because the speech involved is ultimately private, and because the restriction of this speech has severe negative consequences on the judicial system.

Compelled Speech[edit | edit source]

West Virginia State Board of Education v Barnette[edit | edit source]

Facts: In 1942, WVSBE adopted a resolution that compelled all children to salute the American flag.

Rules: A state may not compel individuals to engage in involuntary expression.

Reasoning: Government may censor ideas only if their expression constitutes a clear and present danger of injury. If government censorship is limited, then compulsion should be even more so. Since the students’ refusal to salute the flag didn’t violate anyone’s rights, then the government would not be allowed to compel it. If there is any fixed star in our constitutional constellation, it is that no official, high or petty, can prescribe what shall be orthodox in politics, nationalism, religion, or other matters of opinion, or force citizens to confess by word or act their faith therein.

The Court found that the first amendment cannot enforce a unanimity of opinion on any topic, and national symbols like the flag should not receive a level of defense that trumps Constitutional protections.

The Court followed this principle in other cases, such in Wooley v Maynard, where it ruled that an individual could be punished for blocking out the portion of his car license plate that contained the state motto. The Court said that the right of freedom of speech includes the right to both speak freely and to refrain from speaking at all.

National Federation of Family and Life Advocates v Becerra[edit | edit source]

Facts: NIFLA sought to enjoin the enforcement of the California Reproductive Freedom, Accountability, Comprehensive Care, and Transparency Act. The law’s stated purpose is to ensure access to reproductive health services for California women regardless of income. NIFLA argued that the act’s requirements that (1) licensed clinics provide information to patients about free and low-cost publicly funded services and that (2) unlicensed clinics inform patients of their unlicensed statutes is a violation of their first amendment rights.

Rules: Content-based restrictions on professional speech are subject to strict scrutiny.

Reasoning: Content-based regulations of speech violate the first amendment unless the satisfy strict scrutiny; these means that the regulations must be showed to be properly tailored to serve compelling state interests. The Court never recognized professional speech as a different form of public speech subject to different rules. However, there are two exceptions from strict scrutiny.

  1. When professionals are required to disclose factual, noncontroversial information, in their commercial speech.
  2. Where states regulate professional conduct that incidentally involves speech.

The first requirement did not fit the state’s compelling reason to regulate speech. While the second one was unjustified and unduly burdensome.

Laws prohibiting discrimination[edit | edit source]

Many state and local governments have adopted laws that prohibit discrimination by private groups and clubs. Frequently, those wishing to discriminate bring challenges to these laws; the claim is that freedom of association protects their right to discriminate and exclude whomever they want from their group.

Roberts v US Jaycees[edit | edit source]

Facts: USJ is a social organization that only permitted young men to become regular members. The Minnesota legislature made a law banning these discriminations.

Rules: A state may prohibit a private organization from excluding members on the basis of gender upon showing a compelling interest in preventing these discriminations.

Reasoning: The bill of rights protects the right to form personal relationships without interference from the state. But it doesn’t protect all relationships. Relationships protected by the first amendment are marriage and cohabitation with relatives, which are distinguished by small circles, high selectivity, and seclusion from others. A business doesn’t meet these factors.

A relationship is constitutionally protected depending on

  1. Size
  2. Purpose
  3. Policies
  4. Selectivity
  5. Congeniality

And other pertinent characteristics.

Because the Jaycees were neither small nor selective and participation of strangers was central. It lacked the distinct characteristics needed for constitutional protection. The first amendment protects collective efforts on behalf of shared goals, such as pursuits of political, educational, and cultural ends. Government interference with the internal organization of a group may be justified by compelling interests that can’t be achieved by less restrictive means.

Boy Scouts of America v. Dale[edit | edit source]

Facts: The BSA revoked the membership of one of its eagle scout members because that person was both gay and a gay’s rights activist.

Rules: Under the First Amendment’s protection of the freedom of expressive association, a state may not prohibit a private organization from barring homosexuals from membership.

Reasoning: Forced inclusion of an unwanted person in a group infringes on the group’s freedom of expressive association if the presence of that person significantly affects the group’s ability to advocate public or private viewpoints. However, this freedom is not absolute. It may be override by “regulations adopted to serve compelling state interests, unrelated to the suppression of ideas. The BSA is a nonprofit organization that seeks to instill ethical values in boys. It is indisputable that an association that seeks to transmit such values engages in expressive activity. Thus, it is necessary to determine whether the forced inclusion of Dale would significantly affect that organization’s ability to advocate their viewpoints. The BSA teaches boys to be “morally straight” and “clean.” The BSA argues that homosexuality is at odds with those goals. Just as considerable deference is given to an association’s assertions regarding the nature of its expression, it is also important to give deference to an association’s view of what would impair its expression.

Dale’s membership with the BSA would, force the organization to accept gay people and their conduct. The application of the NJ law would require to accept Dale, which apparently is against the BSA’s viewpoints.

Incorporation[edit | edit source]

Pre-Civil War[edit | edit source]

Individual rights were explicitly protected prior to the Bill of Rights.

Barron v Baltimore[edit | edit source]

This 1833 case limited the reach of the 5th Amendment only to the federal government. Marshall found that the limitations on were specifically intended to limit national government power. He cited the intent of the framers and the development of the bill of rights as an exclusive check on the federal government, Marshall reasoned that the Supreme Court had no jurisdiction in this case because the 5th Amendment was not applicable to the states.

Dread Scott Case – distinguishes between state and national citizenship.[edit | edit source]

  • Scott was held not to be a Us citizen, so no right to use under the 14th amendment.
  • Court held that slaves were property. Yikes
  • Court held that Missouri Compromise was unconstitutional limit on property ownership.

Incorporation of the Bill of Rights into the Due Process Clause[edit | edit source]

Because the Slaughter-House cases, the application of the Bill of Rights to the states could not be through the privileges and immunities clause. In the early twentieth century, the Supreme Court suggested an alternate approach: finding that at least some of the Bill of Rights provisions are part of the liberty protected from state inference by the Due Process clause of the Fourteenth Amendment.

In Chicago, Burlington & Quincy Railroad Co. v. City of Chicago (1897), the Supreme Court ruled that the due process clause of the fourteenth amendment prevents states from taking property without just compensation. Although the Court did not speak explicitly of the Fourteenth Amendment incorporating the Takings Clause, that was the practical decision.

Twining v. New Jersey (1908) is a case where the Court first expressly discussed the incorporation of the bill of rights. The Court said that it is possible that some of the personal rights safeguarded by the first eight amendments against national action may also be safeguarded against state action, because a denial of them would be a denial of due process.

Twinning expressly opened the door to the Supreme Court applying provisions of the Bill of Rights to the states and incorporating them into the Due Process clause.

Palko v Connecticut – Considers the process in context of double jeopardy under Fifth Amendment.[edit | edit source]

  • Selective incorporation debate
  • Is the right “of the very essence of a scheme of ordered liberty” “a principle of justice so rooted in the traditions and conscience of our people to be ranked as fundamental.”

Current State of the Doctrine[edit | edit source]

  • Everything is incorporated except
    • The third amendment right against quartering
    • Fifth amendment right to a grand jury
    • Seventh amendment right to a jury trial in civil cases for more than $20
    • Eighth Amendment right against excessive fines.

Possible Tests[edit | edit source]

  • Palko
    • Implicit in concept of ordered liberty
    • Rooted in traditions and conscience so as to be fundamental
  • Adamson
    • Does the conduct “offend those canons of decency and fairness which express the notion of justice of English-speaking peoples”
  • Duncan v Louisiana
    • Focus on whether the Bill of Rights provision is essential to fundamental fairness.

McDonald v. City of Chicago[edit | edit source]

Facts: Petitioners challenged a law enacted by Chicago that prohibited residents from possessing handguns, claiming the law violated the second amendment

Rules: A bill of rights guarantee applies to the states if it is fundamental to the nation’s scheme of ordered liberty or deeply rooted in the nation’s history and traditions.

Reasoning: The Court found that individual self defense is a basic right, which forms the central component of the Second Amendment’s right to keep and bear arms, and which is deeply rooted in the nation’s history and traditions. Following the civil war, in response to the efforts of some states to disarm black soldiers and other black people Congress enacted a law which protected all citizens’ rights to keep and bear arms.

Economic Substantive Due Process[edit | edit source]

The fifth and fourteenth amendments provide, respectively, that neither the federal nor state governments can deprive any person of life, liberty, or property without due process. The due process clause have been implemented to provide two different types of protection. One is termed “procedural due process” that refers to the procedures the government must go through to take away a person’s life, liberty, or property.

The second is substantive due process which asks whether the government has an adequate reason for taking away a person’s life, liberty, or property. In other words, the focus is on the sufficiency for the government’s action, not on the procedures the government has followed.

Substantive Due Process[edit | edit source]

Depriving people of their fundamental rights requires a substantive due process.

Allgeyer v Louisiana[edit | edit source]

Facts: A Louisiana statute prohibited foreign insurance corporations from conducting business in Louisiana without maintaining at least one place of business and an authorized agent in the state. Louisiana implemented the statute as an exercise of its police power, intending to protect its citizens from deceitful insurance companies. Allgeyer and Company violated this statute by purchasing insurance from a firm based in the state of NY.

Rules: Freedoms protected by the due process clause include economic freedoms and prohibit a state from preventing its citizens from contracting with foreign insurance companies to insure property located within the state.

Reasoning: Although Louisiana would have been able to prohibit corporations from doing business within its borders, the act in question of sending written correspondence to a foreign insurance corporation is permitted by the 14th Amendment. The due process clause of the amendment protects economic freedoms, including the freedom to contract without out-of-state parties. Thus, Louisiana’s attempt to inhibit such contracting, even though it was regarding property located within the state, deprived Allgeyer of its economic liberties without due process.

Lochner v. New York[edit | edit source]

Facts: In 1896, NY legislature enacted the bakershop act which limited the hours bakers were permitted to work to no more than 10 per day. Lochner owned a baker in NY and was fined twice for overworking his employee.

Rules: A state may not regulate the work hours mutually agreed upon by employer and employees as this violates their 14h amendment right to contract freely under the due process clause,

Reasoning: The general right of an employer to make a contract in relation to his business is part of the liberty of the individual protected by the Fourteenth Amendment to the United States Constitution. The right to purchase or to sell labor is part of the liberty protected by this amendment, unless there are circumstances that exclude the right. States may impose reasonable conditions on the right to contract that further the health, safety, and general welfare. Pursuant to their Constitutional police powers, states may prohibit contracts which violate either a federal or state statute, or contracts to use one’s personal property for immortal or illegal purposes.

States have previously been permitted to regulate hours. However, state police power is not absolute and must be balanced against individual liberty concerns protected by the 14th amendment.

Laws protecting unionizing[edit | edit source]

In the early part of the 20th century, as workers attempted to unionize, many states and federal government adopted laws to facilitate unionization by prohibiting employers from insisting, as a condition, that employees do not join a union.

Coppage v Kansas[edit | edit source]

Facts: In 1903, Kansas made it unlawful for employers to prevent their employees from joining a union.

Rule: The due process clause of the 14th amendment prevents states from making laws that prohibit employment contracts barring employees from joining a union.

Holmes Dissent: Lochner should be overruled. The right to contract in that case was wrongly upheld by relying on an economic theory that was largely unsupported by the majority of states that wishes or precedent decisions. This is an issue that should be uniquely decided by each state rather than by the judiciary. There is noting in the Constitution that prohibits states from making laws that one made by Kansas, and thus, the law should be upheld.

Maximum Hours[edit | edit source]

Muller v Oregon[edit | edit source]

Facts: In 1903, the State of Oregon passed a law that limited working hours of female employees to no more than ten per day. Muller, the owner of a laundry business, was convicted for violating the statute after he made his female employees work for more than ten hours.  

Rules: Under the 14th Amendment, a state may constitutionally limit the work hours of men and not women because the state’s strong interest in promoting the health of the weaker sex

Reasoning: Reports, affidavits, and statistics all suggest that working long hours are particularly dangerous to women because of their special physical organization. In addition, to describe the effects of long hours on women’s health, the reports also suggest that working more than ten hours per day outside of the home diminishes women’s ability to perform maternal functions, rear and educate children, and maintain the home.

Minimum Wage Laws[edit | edit source]

Adkins v Children’s Hospital[edit | edit source]

Facts: In 1918, Congress enacted a law guaranteeing a minimum wage to women and children employed in DC’s children’s hospital, which employed many women.

Rules: Congress cannot make a law regulating the federal minimum wage for women as this violates the freedom of contract.

Reasoning: Although the freedom to contract is firmly established in the Court’s jurisprudence, nothing in its precedent decisions suggest that this freedom is absolute. However, the legislature may only interfere with the freedom to contract in the case of exceptional circumstances. While the difference between the sexes was acceptably upheld in Muller, the health and maternal functions concerns that justified that is not present here.

Consumer Protection[edit | edit source]

Weaver v Palmer Bros Co[edit | edit source]

Facts: PA passed the act of 1923 to regulate the manufacturing, sterilization, and sale of bedding. The law specifically prevented the use of ‘shoddy” or scraps of new and second-hand materials, in making new bedding. Palmer uses shoddy in a variety of its products.

Rule: A state may not enact consumer protection legislation when no significant public health and safety concerns exist, or when such concerns may be easily alleviated.

Reasoning: There is no dispute between the parties over the fact that shoddy is capable of being thoroughly disinfected before it is used in the manufacturing of new bedding. Additionally, no reports exist of it spreading sickness or diseases. All health and safety concern can be eliminated through the disinfecting process.

Pressure for Change[edit | edit source]

By the 1930’s, enormous pressures were mounting for the Court to abandon the laissez-faire philosophy of the Lochner era. The depression created a widespread perception that government economic regulations were essential. With millions unemployed  and with wages incredibly low for those with jobs, employees had no realistic chance of bargaining in the workplace.

Lochner rested on the assumption that freedom of contract and related property rights were part of the natural liberties possessed by people. Legal realists attacked this premise and argued that the law reflected political choices, using the freedom of contracts to invalidate state laws was a political choice that favored employers over employees.  

West Coast Hotel Co. v. Parrish[edit | edit source]

Facts: Washington passed a law which regulated the minimum wages paid to female and minor employees. Parrish was employed as a made at a hotel owned by West Coast. Together with her husband, she sued to recover the difference between the wages she was paid and the minimum wage fixed under the law.

Rule: A state may regulate the wage paid to female employees when that regulation is for the purposes of promoting employee’s health, safety, and welfare.

Reasoning: Changing social and economic circumstances since Adkins  warrant a fresh consideration of the issue. The liberty asserted by West Coast is that of freedom to contract, but this freedom is not expressed in the Constitution. Rather, the Constitution, through the 14th amendment, clearly outlines the liberty interests of freedom from actions which attacks a person’s health, safety, and welfare. Thus, all asserted liberty interests are ultimate restrained by the health, safety, and welfare interests that comprise the due process.

changing economic times mean that workers who are not paid a living wage would have to rely on taxpayers for the care of their various needs. This is an unprecedented problem because the United States is currently in the middle of the “Great Depression.” Thus, more workers than ever are seeking community assistance, which leads to an impermissible burden on taxpayer

United States v. Carolene Products Co.[edit | edit source]

Facts: Congress passed the filled milk act that criminalized the shipment in interstate commerce of skimmed milk compounded with any fat or oil other than mil fat, so as to resemble mil or cream. Carolene was accused of shipping Milnut, which was skim milk and coconut oil.

Rules: States posses authority to regulate widely used articles of food for the benefit public welfare.

Reasoning: Congress had a rational basis to enact this law because of the many hearings it conducted to find out whether this would be good for public welfare.

Economic Substantive Due Process Since 1937[edit | edit source]

Williamson v Lee Optical of Oklahoma[edit | edit source]

Facts: In 1955, Oklahoma (OK) made it unlawful for any person not licensed as an optometrist or ophthalmologist in the state to fit lenses to a face or fashion existing lenses into a frame unless given by a prescription by a state licensed eye doctor. Lee Optical sued OK against Williamson, the official charged with enforcing the law, because Lee believed that the law violated the Due process and the equal protection clauses.

Rules: A state may regulate a business if its legislature determines there is a particular health and safety problem at hand and the regulation in question is a rational way to correct those problems.

Reasoning: Although the OK law might be arbitrary and wasteful in many cases, it is absolutely necessary in other cases where directions from a prescription are required for fitting glasses. Regardless, it is a decision for the state legislature, not the judiciary, to balance the pros and cons of the requirement. The legislature made a rational determination that the law is needed in the present case.

Fundamental Rights[edit | edit source]

The supreme court has held that some liberties are so important that they are deemed fundamental rights and that the government generally cannot infringe on them unless strict scrutiny is met.

To meet strict scrutiny, the law must be necessary to serve a compelling governmental interest

  • Ends part of the test –is it a compelling government interest?
  • Means part of the test
  • Does it achieve the purpose/interest?
  • Is it necessary to achieve the purpose/interest:  are there less restrictive alternatives?

The Ninth Amendment[edit | edit source]

This amendment is often mentioned in discussion of fundamental rights, especially rights not expressly mentioned in the Constitution. IT states that the enumeration in the Constitution of certain rights shall not be construed to disparage others retained by the people.

Procedural Due Process[edit | edit source]

The existence of a right triggers two distinct burdens on the government. One is substantive and the other is procedural.

Framework for Analyzing fundamental rights[edit | edit source]

  1. Is there a fundamental right? – If a right is deemed fundamental, the government usually will be able to prevail only if it meets strict scrutiny. But if the rights is not fundamental, generally, only the rational test basis is applied.
  2. Is there a Constitutional right infringed? – If there is a fundamental right, this is the next question. There is no doubt that a Constitutional right is infringed and the government must be justified when the exercise of a right is prohibited.
  3. Is there a sufficient justification for the government’s infringement? – The government needs a compelling state interest to infringe on a fundamental right
  4. Is the means sufficiently related to the purpose? – Under strict scrutiny, it is not enough for the government to prove a compelling purpose, they must also show that the law is necessary to achieve the objective.

Constitutional protection for family autonomy[edit | edit source]

Right to marry[edit | edit source]

Loving v. Virginia – Marriage is one of the basic civil rights of man, it is fundamental to our very existence and survival. To deny this fundamental freedom on so unsupportable basis as the racial classification is surely to deprive all of the state’s citizens of liberty without due process of law.

Zablocki v. Redhail – It is not surprising that the decision to marry has been placed on the same level of importance as the decision to procreate, have kids, raise kids, and family relationships. It would make little sense to recognize a right of privacy with respect to other matters of family life and not with respect to the decision to enter the relation that is the foundation of the family in our society.

(it’s illegal to stop a person from getting married if they have kids)

Bowen v Owens -It is rational to assume that divorced widowed spouses are generally less dependent on the resources of their former spouses than are widows and widowers. Remarriages also lessen these dependencies

Obergefell v. Hodges[edit | edit source]

Facts: In response to some states legalizing same-sex marriage, various states enacted laws and constitutional amendments defining marriage as between one man and one woman. When Obergefell’s partner, John Arthur became terminally ill, they decided to get married in Maryland. After Arthur died, however, the couple’s home state of Ohio refused to list Obergefell as Arthur’s surviving spouse on the death certificate.

April DeBoer and Jayne Rowse adopted 3 kids, but the state banned the adoption of kids by a same-sex couple.

Ipje DeKoe and Thomas Kostura got married in NY before DeKoe deployed to Afghanistan. They later moved to Tennessee which refuses to recognize their union. These and similarly situated plaintiffs all separately sued state officials with enforcing state marriage laws in their respective federal courts, alleging violations of their rights under the Fourteenth Amendment.

Rules: Same sex couples have a Constitutional right to marry. The marriage right must be recognized across all states.

Reasoning: Marriage is a fundamental right under the due process clause. In Loving, the court struck down a law that interfered with the right to marry. In Zablocki, the court invalidated a law that limited the ability of people to get married. Ultimately, the four principles underpinning the protection of the right to marry must apply equally to opposite and same-sex couples

  1. The right to choose whether and whom to marry is inherent in the concept of individual autonomy
  2. The right serves relationships that are equal in importance to all who enter them
  3. Assuring the right to marry protects children and family which implicates a myriad of rights related to procreation and childrearing
  4. Marriage is the very keystone of our social order and foundation of the family unit.

Same-sex couples have an equal right to intimate associations. Refusing to allow same-sex couples to marry denies them a myriad of legal rights, including those related to taxation, insurance benefits, interstate succession, spousal evidentiary privileges, child custody and support, and more. In this instance, the liberty interest protected by the due process intersects with the right to equal protection and same-sex marriage bans violates both.

Right to Custody of One’s Children[edit | edit source]

The Supreme Court has recognized that parents have a fundamental right to custody of their children. The Court has remarked that a natural parent’s desire for and right to the companionship, care, custody, and management of his or her children is an interest far more precious than any property right.

Stanley v Illinois – The Stanleys never married but lived together on and off and had three kids over an 18-year period. When Joan died, the state instituted a dependency proceeding. Under state law, an unmarried father was presumed to be an unfit parent, so they tried to take Stanley’s kids away from him.

The Supreme Court later held that ALL parents are entitled to a hearing to determine their fitness before the state deprives them of custody of their children.

Michael H. v. Gerald D.[edit | edit source]

Facts: Gerald D and Carole D were married in Vegas and resided in California. Carole later cheated on Gerald with a neighbor. Carole gave birth to a kid, Carole then informed Gerald that Michael might be the father. Gerald and Carole split up and Carole took up residence with yet another guy and stopped Michael from seeing his daughter. The child’s guardianship is at issue because a California law presumes that if a mom has a kid during a marriage and her husband is not impotent or sterile, then that kid is the husband’s kid.

Rules: The right of a potential biological father to assert paternal rights over a child born into a woman’s existing marriage with another man is not traditionally recognized in historical jurisprudence and is not a fundamental right protected by the due process clause.

Reasoning: The due process clause jurisprudence governing “liberty” interests requires such interest to be both fundamental as well as traditionally protected by society. An asserted liberty interest of certain parental rights must be rooted in history and tradition. The due process clause of the fourteenth amendment traditionally protects only relationships developed within the unitary family.  No such historical protection exists for a third party, but history is more inclined to protect relationships among traditional family units.

The Right to keep a family together[edit | edit source]

Moore v City of East Cleveland, Ohio[edit | edit source]

Facts: The City of East Cleveland enacted a housing ordinance that limited the occupancy of a dwelling unit to members of a single family. The ordinance narrowly defined the term family as encompassing only a few categories of related individuals. Inez Moore lived in East Cleveland in a home with her son and two grandsons. Under the housing ordinance this arrangement was outside the legal definition of “family”

Rules: The right of related family members to live together is fundamental and protected by the Due Process Clause and necessarily encompasses a broader definition of family than just members of a nuclear family.

Reasoning: The government argued that its housing ordinance should be sustained based on the court’s previous decision in Village of Belle Terre v Boraas where a housing ordinance limiting occupancy in single residence was sustained because it bore a rational relationship to permissible state objectives. However, that case is different because it expressly allowed all who were related by “blood, adoption, or marriage” to live together. It only prevented unrelated individuals from living together, which this one prevents blood relatives from living together. Although the government had a legitimate purpose for the law, it however serves a marginal benefit and is not necessary to accomplish the goal.

The Right of Parents to control the upbringing of their children[edit | edit source]

Meyer v Nebraska (1923)[edit | edit source]

Facts: Under Nebraska law, it was a crime for any individual or teacher in any private, parochial, or public school to teach any subject to any person in any language other than English. Foreign languages could be taught as languages to students only after completion of 8th grade. Meyer, a teacher in Nebraska, was convicted of violating the law by teaching German to Raymond, a child.

Rules; A state may not prohibit the teaching of foreign languages to a young child in school when such teaching has been requested by the child’s parents because this interferes with the fundamental liberty interest of a parent to control his or her child’s education.

SCOTUS ruled the German language may be used as a medium of education in Nebraska.

Reasoning: The scope of the protected liberty interest under the due process clause of the 14th Amendment does not include the freedom from bodily restraint. Rather, recognized liberty interests include the freedom to contract, to engage in any of the common occupations of life, to acquire skill, knowledge, to marry, to establish a home, to raise kids, to practice religion, and to generally the common law notion of happiness. These liberties cannot be interfered with by arbitrary or unreasonable reasons that do not further any legitimate state purpose.

Troxel v. Granville[edit | edit source]

Facts: A Washington statute permitted any person to petition a superior court in the state for visitation rights at any time and authorized the court to grant such visitation rights whenever visitation might serve the best interest of the child. Jenifer and Gary Troxel petitioned a Washington Superior Court for the right to visit their paternal grandchildren after their son, the children’s father, committed suicide. Tommie Granville, the mother of these kids, opposed the petition.

Rules: Under the Due Process Clause, a state court may not grant visitation rights to a person, even when doing so would be in a child’s best interest, if those visitation rights are opposed by the child’s parent because doing so interferes with the parent’s fundamental liberty interest in raising their child.

Reasoning: There is difficulty in attempting to define the concept of average American family because the composition of families varies across the board. Many states have passed similar non-parental visitation statutes to that in Washington, because they recognize the reality that grandparents and other relatives play significant roles in the rearing of children.

Right to Procreate[edit | edit source]

Buck v. Bell – A Virginia law provided that a person with a mental illness could be sterilized for the benefit of the person or society. Sterilization decisions were made by the mental hospital in which the person resided. The court held here that the right to reproduce is not an identified fundamental liberty interest, and thus, the sterilization is allowed.

Skinner v. Oklahoma[edit | edit source]

Facts: An Oklahoma statute, the Habitual Criminal Sterilization Act, allows the forced sterilization of any habitual criminal within the state. The statute defines a habitual criminal as a person, having been convicted two or more times for crimes “amounting to felonies involving moral turpitude” either in Oklahoma Court or in a court of any other state, is thereafter convicted of such felony in Oklahoma and sentenced to prison. That person is then sterilized.

Rules: A state law requiring forced sterilization of criminals convicted of crimes of moral turpitude unconstitutionally infringes on the fundamental rights of marriage and procreation and violates the equal protection clause of the fourteenth amendment.

Reasoning: The legislation deals with some of the most basic civil rights of man. Marriage and procreation, both rights impacted by forced sterilization, are fundamental to the very survival of the human race. The power to sterilize, if exercised, can have a far-reaching and potentially devastating effects not only on Skinner as an individual, but on the future of the human race if done imprudently. The Oklahoma statute infringes on these fundamental rights, so it is best examined with strict scrutiny.

Right to purchase and use contraceptives[edit | edit source]

Griswold v. Connecticut[edit | edit source]

Facts: Griswold was Executive director of planned parenthood. Buxton was a licensed doctor and professor at Yale medical who served as director of the league in New Haven. The center was open and operated from November 1 to 10 of 1961, when the two men were arrested for giving information, instruction, and medical advice to married people for preventing conception.

Rules: An implied right of privacy exists within the bill of rights that prohibits a state from preventing married couples from using contraception.

Reasoning: A right of privacy protecting the intimate relations of married couples is implied in the bill of rights. The ninth amendment provides that the enumeration of rights in the constitution should not be construed to disparage or deny other rights retained by people. The protected activities in each amendments are “penumbras” that are not specifically enumerated but instead represents various zones of privacy in which the government cannot intrude.

Eisenstadt v. Baird[edit | edit source]

Facts: Under a Massachusetts statute, it is a crime to give away any drug, medicine, instrument, or article whatever for the prevention of conception, with the exception of a registered physician providing such items to married couples. Baird was convicted of delivering a lecture on contraception to a group of students in Boston University, and after he gave a young woman a package of Emko vaginal foam.

Rules: Under the Equal Protection clause, a state may not outlaw distribution of contraception to an unmarried person.

Reasoning: The practical effect of the Massachusetts statute is that only married persons seeking contraception from a registered physician for the purpose of preventing conception can get contraception. Single people cannot, and if married people don’t want to prevent a pregnancy but want to get contraception anyway, they’re also barred. The argument that the primary purpose of the law is preventing premarital sex and the spread of disease is rejected. It is unlikely that Massachusetts actually intended to withhold contraception from unmarried fuckers to punish them with unwanted pregnancy or diseases.

Right to Abortion[edit | edit source]

Roe v Wade (1973)[edit | edit source]

Facts: Article 1196 of the Texas Penal Code restricts legal abortions to those procured or attempted by medical advice for the purpose of saving the life of the mother. Roe, a pregnant single woman, brought the suit against a Texas state official on the grounds that the statute was unconstitutional in restricting her right to an abortion.

Issue: Does the Constitutional right to privacy protect a woman’s right to choose to have an abortion

Rule: The constitutional right to privacy protects a woman’s right to choose to have an abortion.

Holding: Yes

Reasoning: The Constitutional right to privacy protects a woman’s right to choose to have an abortion. However, abortions may be regulated by a state after the first trimester of pregnancy and may be completely prohibited after the point of validity of a fetus unless necessary to preserve the health of the mother. Historically, women have had a greater right to terminate their pregnancies than they currently enjoy. There are three reasons to why there was a gradual strictness. First, It was seen to decrease illicit sexual activity by limiting abortion clinics, second concerns over safety of abortion procedures prompted a decrease in its prevalence to protect women’s health, and finally, states increasingly pushing their own interests.

The Court here held that a woman’s right to choose is under the purview of the right to privacy. But, it is not absolute. At some pint in the pregnancy, fetus being seen as a person outweighs the mother’s privacy. Thus the state interests grow substantially. With respect to the state’s interests in protecting the health of the mother, the interests becomes compelling at approximately the end of the first trimester. A state can therefore only regulate after viability, not before, but when a mother’s life is in danger, then she may have an abortion.

Planned Parenthood v Casey[edit | edit source]

Facts: Planned parenthood brought a suit against Case, the governor of PA, challenging five restrictions on abortion under PA law. Most significantly, the PA statute required informed consent and a 24-hour waiting period for all women prior to undergoing an abortion. All minors seeking abortion were required to obtain the informed consent oof at least one parent, while a married woman need to show that she informed her husband of her intent to deletus fetus.

Issues: Is a state court restriction on abortion that requires informed consent and a 24-hour waiting period unconstitutional?

Rules: A state abortion regulation places an undue burden on a woman’s right to an abortion and is invalid if its purpose or effect is to place a substantial obstacle in the pat of a woman seeking an abortion before the fetus attains viability.

Holding: No

Reasoning: The holding in Roe v Wade is reaffirmed. There, the Court held: (1) a woman has the right to choose to have an abortion before viability to obtain it without undue interference from the state (2) a state may restrict abortions after fetal viability as long as it passes a law that exempts pregnancies that endanger the woman’s life or health and (3) a state has a legitimate interest from the outset of the pregnancy in protecting the health of the woman and the life of the fetus. The precedent holdings should be overturned only if changing circumstances render the established rules unworkable. Although Roe has endangered opposition, it has in no sense proven unworkable in its limitations of state restrictions on abortion. Although medical advances have moved the moment of viability, the holding in Roe is still relevant. The PA restrictions in its present case can be upheld as constitutional regulations on abortion designed to help women make informed and rational choices. An undue burden exists because it puts a substantial obstacle that interferes with a woman’s right to an abortion.

Government Regulation of Abortions[edit | edit source]

Whole Woman’s Health v Hellerstedt[edit | edit source]

Facts: The state of Texas passed two laws governing abortions. The first required that a doctor performing the abortion have admitting privileges at a hospital no more than 30 miles from where the abortion is going to be performed (Admitting-privileges Requirement). This provision was adopted to ensure that women had easy access to a hospital in the event that complications from the abortion arose. The second provision required that the standard for each abortion facility meet the minimum standards for ambulatory surgical centers (The surgical center requirement). Whole Woman sued the defendant, commissioner of the Texas Department of State Health Services, claiming that the laws were unconstitutional. The district court determined that the laws would reduce the number of abortion facilities in Texas from 40 to eight.

Rules: A law with the purpose or effect of placing a substantial obstacle in the path of a woman seeking an abortion imposes an undue burden on a woman’s right to have an abortion and is thus unconstitutional.     

Holding: Yes

Reasoning: A law with the purpose or effect of placing a substantial obstacle in the path of a woman seeking an abortion imposes an undue burden on a woman’s right to have an abortion and is unconstitutional. Under Planned Parenthood v Casey, states can impose limited restrictions on abortions that ensures the safety of the patient, but these restrictions cannot unduly burden the patient’s right to have an abortion. In this case, the district court did not err that the laws were Unconstitutional.

Gonzales v Carhart[edit | edit source]

Facts: In November 2003, President Bush signed into law the Partial-Birth Abortion Ban Act. The PBABA prohibited “intact dilation and evacuation” (intact D&E), a particular manner of ending fetal life in the second trimester whereby a surgeon killed the fetus by dilating the mother’s cervix, and pierce the fetus’ head with scissors, and using suction to extract the fetus from the Uterus. Dr. Leroy Carhart, a physician that performed this procedure brought suit in federal district court against AG Alberto Gonzales based on the Constitutionality of PBABA.

Rules: Congress may ban a specific type of partial-birth abortion provided its restrictions on the practice are narrow and clear and the ban does not constitute an undue burden on a woman’s right to an abortion

Reasoning: Before viability, a state cannot prohibit any woman from getting an abortion. Additionally, a state cannot impose upon this right an undue burden. An undue burden exists if the purpose or effect of a regulation is to place a substantial obstacle in the path of a woman seeking an abortion before viability. However, a regulation that merely creates a structural mechanism in which the state, or the parent, guardian or a minor, can express profound respect for the life of the unborn is permitted if the regulation is not a substantial obstacle.

Constitutional Protection for Medical Care Decisions[edit | edit source]

Right to refuse[edit | edit source]

Washington v Harper – The Court said that prisoners had the right to be free from involuntary administration of antipsychotic drugs. The court observed that prisoners possess a significant liberty interest in avoiding unwanted administration of antipsychotic drugs under the Due Process clause. The court therefore said that the force administration of the medication in a nonconsenting person represents a substantial interference with that person’s liberty.

Cruzan v. Director, Missouri Department of Health[edit | edit source]

Cruzan (1990) is the first SCOTUS "right to die" case.

Facts: Nancy Cruzan was involved in a serious automobile accident. Paramedics found Cruzan without respiratory or cardiac functions, but revived her at the scene. After waking up from a coma, she stayed in a vegetative state in which she exhibited no cognitive or motor functions. To assist her, surgeons implanted a gastronomy feeding and hydration tube. When it became clear that she was not going to recover, her parents opted to pull the plug.

Rules: A state may require that a guardian seeking to remove life-prolonging treatment prove by clear and convincing evidence that the person in the persistent vegetative state would have wanted the treatment withdrawn under such circumstances.

Reasoning: Based on precedent, a competent person has a constitutionally protected liberty interest under the Fourteenth Amendment to refuse unwanted medical treatment. However, an incompetent person is unable to make such a choice. A choice must be made for them by their guardian. Thus, a court must balance the liberty interests of the incompetent individual and the state’s interest in preserving life.

Right to Physician assisted death[edit | edit source]
Washington v Glucksberg[edit | edit source]

Washington v Glucksberg (1997) is another major right-to-die SCOTUS case.


Facts: Under Washington state law, it is a crime to knowingly cause or aid another person to attempt suicide. Glucksberg, a Washington physician, along with other doctors sued the state alleging that Glucksberg frequently treated terminally-ill patients and would have assisted those patients in ending their lives if not for Washington’s ban on assisted suicide. Glucksberg brought suit in federal district court seeking a declaration that the Washington state law violated a liberty interests protected by the Fourteenth Amendment.

Rules: The right to physician-assisted suicide is not a constitutionally-protected liberty interest under the Due Process Clause of the Fourteenth Amendment.

Reasoning: In almost every state, there is a ban on physician-assisted suicide, representing those states commitment to the protection and preservation of all human life. Even in more recent years with advances in medical technology, most assisted-suicide bans have been reexamined and reaffirmed by states. Even though many states have expanded their laws to permit living wills and surrogate decision-making regarding the withdrawal of life sustaining medical equipment, voters and state lawmakers alike consistently uphold bans on assisted suicide.

There is a two step analysis on whether a fundamental right exists under the due process clause

  1. Whether the right is objectively rooted in the US history and tradition and implicit in the concept of ordered liberty such that neither liberty nor justice would exist if that right is sacrificed
  2. Whether careful description exists of the fundamental liberty.

Constitutional protection for sexual orientation and sexual activity[edit | edit source]

Bowers v Hardwick – The Supreme Court held that the right to privacy does not protect a right to engage in private consensual homosexual activity. The Court opined that it was not a right that exist because it was not supported by the constitution’s text, the framer’s intent, or tradition.

Lawrence v. Texas[edit | edit source]

Facts: Cops were brought to the home of John Lawrence in response to a report of a weapons disturbance. When the police went into his home, they say him engaging in gay love with his partner. The state charged the two men with engaging in deviate sexual intercourse with a person of the same sex.

Rules: The due process clause does include a right to liberty in a person’s decisions concerning intimacies of their physical relationships.

Reasoning: The due process clause does include a right to this liberty. The right to liberty under this clause has been held to protect the rights of married couples to make decisions regarding procreation by invalidating a law prohibiting the use of contraception. His same right has been extended to unmarried couples. This right was the basis of the Roe v Wade decision which held that a woman has the right to elect an abortion under some circumstances as a fundamental decision affecting her future. This right to liberty has been extended to decisions relating to marriage, procreation, contraception, family relationships, child rearing, and education. However, Bowers v Hardwick upheld a law that goes against this notion. The decision’s reliance on historical data and practices were  misplaced. Much of the information relied on by Bowers has been put into question. Additionally, since the decision was issued, there has been an emerging awareness that liberty gives substantial protection to adults in deciding how to conduct their private lives in matters relating to sex.

Regardless of historical circumstances, the key issue is whether the majority may use the power of the state to enforce its views of morality on the whole society through operation of criminal law. The answer is no.

Right to Education[edit | edit source]

San Antonio Independent School District v Rodriguez[edit | edit source]

Facts: Mexican-American parents whose children attended schools in Edgewood Independent School district brought a class action lawsuit against the school district. The suit was brought on behalf of school children throughout the state who were members of minority groups or who were poor and resided in school districts financed by a low property tax base. In the late 40’s, the Texas legislature sought to mitigate the inequality of resources among school districts created by differences in property tax bases. The legislature enacted the Texas Minimum Foundation school Program, which called for state and local contributions to a fund earmarked specifically for teacher, salaries, operating expenses, and transportation costs. Individual school districts were responsible for providing twenty percent of the revenue for this fund and did so by imposing property taxes on citizens residing within the district. The property values in Rodriguez’s district were far lower than property value of other districts, making the amount collected to educate Rodriguez’s kids significantly lower per pupil than that allocated for children in more affluent districts.

Rules: Education is not recognized as a fundamental right under the fourteenth amendment to the constitution, and thus a state regulation impacting the right to education should be analyzed under rational basis review to determine if it bears a rational relationship to a legitimate state purpose.

Reasoning: The analysis on whether education itself is explicitly or implicitly guaranteed as a right in the Constitution. It is not explicitly written and there is no implicit basis for holding education is so protected. Rodriguez’s argument that education is essential to the effective exercise of First Amendment freedoms and to intelligent utilization of the right to vote is rejected. No evidence has been offered that the present levels of educational expenses in Texas provide an education falling short of fulfilling these purposes.

Constitutional Provision concerning equal protection[edit | edit source]

The Constitution as originally drafted and ratified had no provisions assuring equal protection of the laws. This of course, is not surprising for a document written for a society where blacks were slaves and where women were discriminated against. After the Civil war, widespread discrimination against former slaves led to the passage of the Fourteenth Amendment, which provides in part that no state shall deny a person within its jurisdiction the equal protection of the laws.

A framework for equal protection analysis.[edit | edit source]

What is the classification –[edit | edit source]

The first question is what is the government’s classification? How is the government drawing a distinction among people? Equal protection analysis must always begin by identifying how the government is distinguishing among the people.

What is the appropriate level of scrutiny?[edit | edit source]

Discrimination based on race or national origin is subject to strict scrutiny. Also, generally, discrimination against aliens is subject to strict scrutiny. Although, there are several exceptions where less scrutiny is used.

The government must have a compelling government purpose

The government must have a significant reason for discriminating

It must show that it cannot achieve its objective through less discriminatory means.

The government has the burden of poof

Intermediate scrutiny is used for discrimination based on gender and against nonmarital children. Under this level, a law upheld if it is substantially related to an important government purpose. The means used need not be necessary, but must have a substantial relationship to the end being sought.

Rational basis is a level of scrutiny that all laws challenged under equal protection must meet. All laws not subject to the higher two levels are evaluated here. Here, a law will be upheld if it is rationally related to a legitimate government purpose. The objective need not be compelling or important. The means chosen only needs to be a rational way to meet the end.

The challenger has the burden of proof for a rational basis test

Does the government action meet the level of scrutiny? –[edit | edit source]

The level of scrutiny is the rule of law applied to a particular government action that’s challenged. The court evaluate both the law’s ends and its means.

In evaluating the relationship of the means of a particular law to the end, the Court often focuses on the degree to which a law is underinclusive or over inclusive.

And underinclusive law is one that does not apply to individuals who are similar to those whom the laws applies

Overinclusive laws are ones that, if it applies to those who need not be included in order for the government to achieve its purpose.

The protection of fundamental rights under equal protection[edit | edit source]

Usually equal protection is used to analyze government action that draws distinction among people based on specific characteristics. Sometimes, though, equal protection is used if the government discriminates against people as to the exercise of their fundamental rights.

Skinner v Oklahoma[edit | edit source]

Rules: A state law requiring forced sterilization of criminals convicted of crimes of moral turpitude unconstitutionally infringes on the fundamental rights of marriage and procreation and violates the equal protection clause of the fourteenth amendment.

Reasoning: The legislation deals with some of the most basic civil rights of man. Marriage and procreation, both rights impacted by forced sterilization, are fundamental to the very survival of the human race. The power to sterilize, if exercised, can have a far-reaching and potentially devastating effects not only on Skinner as an individual, but on the future of the human race if done imprudently. The Oklahoma statute infringes on these fundamental rights, so it is best examined with strict scrutiny.

The rational basis test[edit | edit source]

What constitutes a legitimate purpose?[edit | edit source]

Romer v Evans

Facts: Several Colorado municipalities passed ordinances banning discrimination based on sexual orientation in housing, employment, education, public accommodations, health, and welfare services. In response to this ordinance, Colorado voters passed Amendment 2, which prohibited all governmental intervention designed to protect the status of a person based on their sexual orientation, conduct, practices, and relationship. Evans represented an aggrieve class of homosexuals who sued the state governor.

Rules: A law prohibiting anti-discrimination protections for the gay, lesbian, and bisexual community violates the Equal Protection Clause of the Fourteenth Amendment.

Reasoning: The equal protection clause provides that no person may be denied the equal protection of the laws. If a law neither burdens a fundamental right nor targets a suspect class, the law will pass Constitutional muster so long as it is rationally related to a legitimate state purpose. In this case, Colorado argues that Amendment 2 is not unconstitutional because it puts the LGBTQ+ in the same spot as all other persons. However, the state supreme court found that the effect of Amendment 2 repeals existing statutes, regulations, and policies that bar discrimination based on sexual orientation. Additionally, it functions to ensure that no similar laws protecting the LGBTQ will ever be enacted. Thus, the amendment does treat them differently from the other people. Thus, it does not have any legitimate purpose to pass the rational basis test.

Must it be the actual purpose or is a conceivable purpose enough?

The Court’s enormous judicial deference under the rational basis test is, in part, because of its willingness to accept any conceivable legitimate purpose as sufficient, even if it was not the government’s actual purpose.

The requirement for a reasonable relationship

Under the rational basis review, the Court must also decide whether the classification drawn in a statute are reasonable in light of its purpose. However, the Court repeatedly has expressed that this is the most relaxed and tolerant form of scrutiny.

Tolerance for under inclusiveness under rational basis review.

Railway Express Agency v New York

Facts: A law made by the state of NY prohibited vehicles devoted solely to displaying advertisements, but permitted business vehicles to display signs related to their business so long as the vehicles were not solely used for advertising.

Rules: A state law that is substantially underinclusive does not necessarily violate the Equal Protection Clause because a state may rationally decide to address public problems in phases

Reasoning: In passing the law, NY stated that it was seeking to address a traffic congestion problem. Railway argued that to be rationally related to this legitimate purpose, the law should have been regulating trucks operating in NY and not the contents of the advertising on those trucks. It does not matter that the regulation seems under inclusive, as the regulation could have rationally decided that it is most pressing to regulate just one type of advertising at that time.

Tolerance for overinclusiveness

New York Transit Authority v Beazer

Facts: The NYTA made a policy that it would not hire or employ people currently participating in one of NYC’s several methadone maintenance programs. About 40,000 persons received methadone treatments in NY and treatments were primarily administered to treat the physical withdrawal symptoms of heroin addicts. Beazer and three other persons brought suit in federal district court against NYTA in a class action on behalf of all people who had been or would have been subject to discharge or rejection by the NYTA.

Rules: A state regulation that is over inclusive because it regulates a general class of people based on the conduct of particular members within that class does not violate the Equal protection clause of the constitution if it is rationally related to a legitimate purpose.

Reasoning: Beazer argued that users of methadone should not be included in the general class of narcotics users. However, the district court found that several differences exist between users of methadone and persons that used no drugs at all. Additionally, a substantial number of people enrolled in methadone maintenance programs cannot be expected to complete them, and thus there is a great risk of recidivism among employees and potential hires in this class of persons. The NYTA can try to distinguish among methadone users and determine which people are most likely to present a drug risk, yet such a rule would be imprecise and almost impossible to apply in practice.

Cases in which laws are deemed arbitrary and unreasonable

US Department of Agriculture v Moreno

Facts: Congress passed a food stamp act to govern and reform its food stamp program. A part of the act excluded people from participating in food stamps if their household contained people who were unrelated to any other people in the household. Moreno lived with Sanchez, a person to whom she was not related and Sanchez’s three kids. Moreno met all the income requirement, but did not receive any benefits because of the household issue.

Rules: A state regulation that arbitrarily creates two classes of persons and deprives one class of government benefits violates the Equal protection and due process clause because it is based on mere legislative preference for one class that is not rationally related to a legitimate state purpose.

Reasoning: For this legislative classification to be upheld, it would have to be rationally related to a legitimate governmental interest. In the present case, Congress stated that the purpose was to stimulate the agricultural economy by encouraging people to purchase farm surpluses. The governing of relationships within private homes bears no rational relation to the state purpose. However, because legislative enactments do not necessarily have to be sustained by the actual purpose, an analysis is undertaken to see if other purposes exist that can constitutionally sustain the statute. Congress argued that unrelated people in houses are more likely to abuse the program and thus could have passed legislation that is rationally related to the legitimate purpose of preventing fraud. However, this act did not target people likely to abuse the program or commit fraud, but rather it targeted people that are so desperately need of aid that they cannot afford to alter their living arrangements for program eligibility.

City of Cleburne, Texas v. Cleburne Living Center, Inc.

Facts: The living center filed an application for a special use permit with the city. The center should a permit to build residential housing for mentally disabled people. The facility would house up to thirteen people, who would be supervised at all times. The city denied the application.

Rules: The mentally disabled are not a quasi-suspect class and thus any legislative regulations affecting their rights are subject to rational basis review and not intermediate scrutiny.

Reasoning: The mentally disabled are not a quasi-suspect class and thus rational review would have been appropriate. First, it is undeniable that mentally disabled people require special care for functioning in the world, so legislative judgments are likely to be rational and should not be criticized more closely. Second, on a national scale, federal lawmakers have shown great appreciation for the plight of the mentally ill and enacted significant laws prohibiting discrimination against them. Third, these laws show that mentally ill people are not politically powerless and have been able to attract attention of lawmakers to provide their rights. Fourth, finding the mentally ill a quasi-suspect class would pose problems in the future for classifying other groups as such that possibly share some but not all characteristics with them. For these reasons, rational basis review is appropriate for any law related to the mentally ill.

However, there was no rational basis for the city to reject the home. They require special permits for this housing plan but not for other building plans. Their goal was supposedly to avoid any harassment or negative attitude towards the mentally ill community, but there was no connection between this and their permit.

Classification based on race and origin[edit | edit source]
Race and discrimination before the thirteenth and fourteenth amendment[edit | edit source]
Dread Scott v. Sandford[edit | edit source]

Facts: Dread Scott was an African American man born a slave in Virginia in the late 1700s. In 1830, he was taken by his owners to Missouri and purchased by Army Major John Emerson in 1832. Emerson took Scott with him on various assignments in areas where slavery was outlawed. While in Wisconsin, Emerson allowed Scott to marry and later left Scott and his wife in Wisconsin when he was reassigned to Louisiana. While in Louisiana, Emerson married Eliza Sandford. He then sent for Scott and his wife to travel to Louisiana to serve him and Eliza. Scott tried to buy his freedom, but Eliza refused.

Rules: People of African descent brought to the United States and held as slaves, as well as their descendants (either slave or free), are not considered citizens of the United States and are not entitled to the protections and rights of the Constitution.

Reasoning: The inquiry is necessarily limited to a class of people that include only those whose ancestors were African, imported and sold in the US as slaves. AT the time of the framers’ drafting of the Constitution, the prevailing view in the United States was that African people were slaves and therefore property and not people.

Post Civil War Amendments[edit | edit source]

Strict Scrutiny for Discrimination based on race and national origin

It is now clearly established that racial classification will be allowed only if the government can meet the heavy burden of demonstrating that the discrimination is necessary to achieve a compelling government interest. Ironically, the Court first articulated this requirement in Korematsu v Japanese Americans during World War II. The Court held that all legal restrictions that curtail the civil rights of a single racial group are immediately suspect. That is not to say that all restrictions are unconstitutional, but must be subject to the most rigid scrutiny.

Proving Existence of a Race or national origin

There are two alternative ways of demonstrating the existence of a race or national origin classification.

Is where the classification exists on the face of the law

Alternatively, if a law is facially neutral, a race or national origin classification might be proven by demonstrating discriminatory administration or impact.

Race Specific Classifications that disadvantage racial minorities.

Korematsu v United States

Facts: On May 9 1942, under Civilian Restrictive Order No 1, based on Executive Order 9066, Japanese-Americans were ordered to move to relocation camps in light of the US’s involvement In WWII. Korematsu was an American citizen of Japanese descent who was convicted by the United States Government in federal district court for violating Civilian Order No 34 which excluded Japanese Americans from remaining in San Leandro, a region designated as a military area.

Rules: State laws restricting the rights of persons based on race are subject to strict scrutiny and will only be upheld if they further a “pressing public necessity.”

Reasoning: Although all legal restrictions which restrict the civil rights of a single racial group is automatically suspect, it does not follow that all such restrictions are automatically unconstitutional. Such restrictions are subject to rigid scrutiny by the courts and will only be upheld in instances of pressing public necessity. The United States Government does not have the resources to make individualized determinations of loyalty during the war effort, therefore exclusion of Korematsu from the West Coast, regardless of his personal loyalties, is justified because of the existence of a “pressing public necessity.

Racial Classifications Burdening both whites and minorities

Loving v. Virginia

Facts: Mildred Jeter, a black woman and her husband, Richard Loving, a white man moved to Virginia and resided in Caroline County. The laws of the state banned interracial marriages. They were then indicted for violating Virginia law, where they plead guilty and were sent to jail.

Rules: A state may not restrict marriages between persons solely on the basis of race under the Equal Protection and Due Process Clauses of the Fourteenth Amendment.

Reasoning: State bans on interracial marriages were passed as a reaction to slavery and have been present since the colonial period. Such bans were affirmed by the Racial Integrity act of 1924, passed during a period of extreme nativism following WWI. However, in the fifteen years preceding this case, fourteen states have repealed their own similar bans. In the present, this state seeks to uphold its ban on the grounds that it has a legitimate purpose to preserve racial integrity and pride. It argues that it is complying with obligation by prevent all interracial marriages, not just whites. The argument that the mere equal application is enough to pass constitutional muster is obviously rejected.

Palmore v. Sidoti

Facts: The Sidotis were both white people, married to each other. They had a kid. They later divorced and Linda lived with and married a black man named Clarence Palmore Jr. After learning of this, Sidoti sought sole custody of their kid, citing that changed conditions as grounds for his petitions, as well as making several allegations of instances in which Linda had not properly cared for the child. After the hearing, Florida court made no findings of fact which indicated that the child had not been properly cared for by either parent. However, the curt relied on the recommendation of a counselor and awarded custody to Anthony because Linda was married to a black man.

Rules: The deprivation of custody of an infant from its mother solely because of the risk of racial biases violates the Equal Protection Clause.

Reasoning: Child custody in state court proceedings are rarely in the purview of the federal judiciary. The lower court’s expressions of its views about the damaging effect on the child from living in a racially mixed household, as in this case, raises some federal issues. Racial and ethnic prejudices are still unfortunately alive and well, and the child in the case might very well be subject to these biases. However, the existence of these biases is not enough to remove an infant from its mother’s custody.

Segregation Laws

Plessy v Ferguson

Facts: In 1890’s, the state of Louisiana passed a law that provided for the separate railway cars of whites and blacks. Plessy, a biracial man, challenged the law by taking a seat in the white railway. He was forcibly ejected and jailed.

Rules: Public accommodation that are segregated according to racial classifications do not violate the equal protection clause so long as such accommodations are separate but equal.

Reasoning: While the object of the Fourteenth Amendment is to promote the equality of all races before the law, it cannot have been intended to abolish all distinctions based on color, or to enforce social equality when whites and blacks do not actually want to commingle. These laws are fine so long as they do not implicate superiority or inferiority.

Attacks on the Separate but equal laws

Missouri ex rel Gaines v Canada ([1]) –

The Supreme Court held that it was unconstitutional for Missouri to refuse to admit black kids to its law school, but instead to pay for blacks to attend out-of state law schools. The Court explained that the basic consideration is not as to what sort of opportunities other states may provide, but as to what opportunities the state itself furnishes to white students and deny to black students solely based on color. In response, the state made new school for black students instead of admitting them into the white schools.

Sweatt v Painter – Sweatt

was a black person who applied to a Texas law school. However, the state’s black only law school offered to admit Sweatt when the whites only school denied him. He sued the rejecting university on racial grounds. . No matter how comparable the two schools might be in resources or other tangible factors, they are not comparable in the opportunities they would provide for Sweatt to network with the white students who constitute the majority of future Texas lawyers or to obtain a prestigious degree that would give him access to top legal jobs. By denying Sweatt equal access to these intangible benefits, Texas violated the Equal Protection Clause.

McLaurin v Oklahoma

The Supreme Court held that once blacks were admitted in a previously all-white school, the university could not force them to sit in segregated areas.

Brown v Board of Education

Facts: The present case represented a consolidation of several states where African American minors sought the aid of their state courts in gaining admission to public schools on a non-segregated basis. In all instances, Brown, and other minor African American children were denied admission to public schools attended by white children under laws requiring or permitting segregation of race. They alleged that the segregation deprived them of the equal protection of the laws under the fourteenth amendment.

Rule: Separate educational facilities based on racial classifications are inherently unequal and violate the Equal Protection Clause of the Fourteenth Amendment.

Reasoning:  In deciding the issue, it is impossible to rely on the original intent surrounding adoption of the Fourteenth Amednment because prior cases and the legislative history involved in its enactment are inconclusive as to the true extent of its meaning. Additionally, it is not helpful to look at the status of public education at the time the amendment was adopted.

The  basic language of the amendment suggests that it was passed to prohibit all forms of discrimination against African Americans. In the present case, all basic attributes of the white and black schools are essentially the same. To determine whether the segregated schools violate the Fourteenth Amendment as interpreted, it is necessary to examine the actual effect of this segregation on the institution of public education as a whole. Modern studies confirm that the children experiencing segregation feel inferior, become less motivated, and perform at a lower standard than children that do not experience segregation.

Facially Neutral Laws with Discriminatory Impact or Administration[edit | edit source]

The requirement of proof of a discriminatory purpose.

Washington v. Davis

Facts: Davis was an African American who, along with another black person applied for admission to the DCPD. Both men were turned down and brought suit in federal district court against Washington. , the mayor of Washington, D.C., alleging that the police department used racially discriminatory hiring practices by administering a verbal skills test (Test 21) disproportionately failed by African Americans.

Rules: A state-sponsored racial classification violates the equal protection provisions in the Fifth Amendment’s Due Process Clause only if it is shown to have both a disproportionate impact on a particular race and is motivated by invidious racial discrimination.

Reasoning: The purpose of the equal protection clause is to prevent official conduct that discriminates on the basis of race. However, the Court has never adopted a rule which invalidates official conduct that merely has disproportionate impact on a particular race without evidence of a discriminatory purpose. However, this purpose can be inferred based on the totality of facts, including the fact that a law burdens one race more heavily than another. However, the mere instances of a disproportionate impact does not, without more, trigger strict scrutiny.

The test itself is neutral on its face and was administered to all applicants to ascertain whether they had reached a particular level of verbal skill necessary for becoming a successful police officer. Thus Washington, D.C. did not act unconstitutionally in its hiring practices. The judgment of the court of appeals is reversed.

McCleskey v. Kemp

Facts: McCleskey is a black man who was convicted of two counts of armed robbery and one count of murder in Atlanta. At trial the jury recommended that he be sentenced to death on the murder charge. The court followed the recommendation. He filed a petition for a writ of habeas corpus in federal district court, alleging that the sentencing was administered in a discriminatory manner.

Rules: A criminal defendant alleging an equal protection violation must prove the existence of a discriminatory purpose and a racially disproportionate and discriminatory effect.

Reasoning: He must prove that the decisionmakers in this case acted with a discriminatory purpose. He offers no evidence to prove his claim, relying entirely on study results. IF the study were accepted as evidence, then an equal protection violation would occur in every instances in which a black man was sent to death for murdering a white man.

Is proof of a discriminatory effect also required[edit | edit source]

Palmer v Thompson

Facts: In 1962, Jackson Mississippi operated 5 public parks, four of the swimming pools were designated for whites only. Fifth pool was designated for blacks only. .

Rules: A state regulation based on racial classifications is unconstitutional under the Equal Protection Clause if the plaintiff proves both a discriminatory purpose and a discriminatory impact of the regulation.

Reasoning: Neither the Fourteenth Amendment nor any act of Congress places an affirmative duty on states to operate swimming pools. Additionally, this case does not present a situation where whites are permitted to use pools while blacks are denied access. None of the Court’s precedents invalidate legislative judgements solely based on discriminatory intent. Discriminatory intent is difficult to prove because of the many viewpoints and compromises factored into a legislative decisions. However, the plaintiff here failed to prove any discriminatory intent or impact.

How is a discriminatory purpose proven?[edit | edit source]
Personnel Administration of Massachusetts v Feeney[edit | edit source]

Facts: The state enacted a veteran preference statute that provided that all veterans qualified for state civil service positions must be considered for appointment ahead of any qualifying non—veterans. Feeney was a female non-veteran applicant for the civil service that scored very high on a number of competitive civil service exams. She was passed over for males with lower scores who were veterans.

Rules: To prove that a state actor violates the Equal Protection Clause by enacting legislation with a discriminatory purpose, a plaintiff must show that the decisionmaker selected or reaffirmed a particular course of action at least in part because of, not merely in spite of, its adverse effects upon an identifiable group.

Reasoning: The state’s veteran preference law was enacted to reward veterans for their service, to ease their transition from military to civilian life, to encourage patriotic service and to attract loyal and disciplined people into the civil service. Despite the state’s attempt to include as many women as possible, a disproportionate amount of men are included largely because of gender-based restrictions in the military itself.

Village of Arlington Heights v. Metropolitan Housing Development Corp.[edit | edit source]

Facts: The Metropolitan Housing Development Corp applied for a permit from the Village of Arlington Heights to rezone a fifteen-acre parcel of land from its zoning classification as a single-family use to a multiple-family use classification. MHDC planned to build a racially-integrated complex featuring nearly two hundred townhouse units marketed to low and moderate income tenants. The village denied the permit request and MHDC brought suit alleging that the denial of the permit was racially charged.

Rules: A state-sponsored racial classification will not be held to violate the Equal Protection Clause of the Fourteenth Amendment unless a plaintiff shows that the law is motivated by a discriminatory purpose and has a discriminatory impact.

Reasoning: In determining the existence of a discriminatory purpose, several factors must be considered

The historical background predating the decision

The specific sequence of events leading up to the challenged classification

Departures by the state actor from normal procedures

Substantive departures, particularly if the factors usually considered important by the decisionmaker strongly point to a decision contrary to the one reached

The legislative or administrative history surrounding the adoption of the legislative classification.

Nothing in the factual records indicated that the sequence of events leading up to the denial of the permit sparks suspicion.

Application: Discriminatory use of peremptory challenges[edit | edit source]

One of the most important areas where the Court has followed and applied this analysis is in holding Unconstitutional the discriminatory use of peremptory challenges. Law providing for peremptory challenges are facially race neutral. But peremptory challenges based on race or gender are motivated by a discriminatory intent and have a discriminatory impact. Those, the Court has held that race or gender based peremptory challenges deny equal protection whether exercised by a prosecutor, criminal defendant, or civil litigant.

Batson

First, the criminal defendant must set forth a prima facie case of discrimination by prosecutor

Second, one the defendant has presented a prima facie case of discrimination, the burden shifts to the prosecutor to offer a race-neutral explanation for the peremptory challenges

Third, the trial court must decide whether the race-neutral explanation is persuasive or whether the defendant has established purposeful discrimination

Introduction: The Problem of Remedies[edit | edit source]
Brown II[edit | edit source]

Facts: In its original decision in Brown v. Board of Education (Brown I), the United States Supreme Court held that racial discrimination in public education was unconstitutional. The Court upheld a challenge by Brown to discriminatory racial policies in public schools operated by various boards of education in several states.

Rules: Adequate compliance with the Court’s previous holding that racial discrimination in public education in unconstitutional requires public schools to desegregate “with all deliberate speed”

Reasoning: Since the original decisions, one year ago, public school have taken substantial steps towards full desegregating. Individual school officials are responsible for implementing constitutional principles in good faith, but the various district court that originally heard the cases against all schools involved in Brown are best equipped to determine whether a good faith effort was made. The case is remanded to those courts. In rendering decisions on the cases, each respective court is guided by principles of equity.

Regents of Univ. of CA v. Bakke[edit | edit source]

Facts: UC, Davis Medical school practiced a policy whereby it reserved 16 out of 100 places in its entering class for members of racial minority groups. A special committee was appointed to administer this admissions policy. Allan Bakke brought suit against the regents of the school on the grounds that the policy was unconstitutional.

Rules:  Under the Equal Protection Clause of the Fourteenth Amendment, a public university may not discriminate on the basis of race in its admissions policies, even if doing so benefits members of minority races.

Reasoning: Under the equal protection clause, a public university may not discriminate on the basis of race in its admissions policies, even if doing so benefits minority races. All racial and ethnic classifications are inherently suspect and must be viewed with strict scrutiny.

Fullilove v. Klutznick[edit | edit source]

Facts: Congress enacted a statute to help minority business enterprises which were businesses owned or controlled by black americans and other minorities. Government reports and studies in the congressional record establish that MBEs had been discriminated against in public contracting opportunities.

Rule: A federal affirmative-action statute that conditions the receipt of federal funds on state and local governments using a portion of the funds to purchase goods or services from minority business enterprises is constitutionally permissible under the Fourteenth Amendment’s Equal Protection clause.

Reasoning: A federal statute that conditions the receipt of federal funds for building projects on state and local governments using at least 10 percent of the funds to purchase goods or services from MBEs is constitutionally permissible. It was held in Bakke that intermediate scrutiny was the appropriate test in such cases. In this case, the statute survives intermediate scrutiny and is upheld.

Wygant v. Jackson Board of Education[edit | edit source]

Court held layoff plans was not necessary to achieve compelling purpose of remedying proven past discriminations as the district could achieve it with less discriminatory means

Court held remedying past societal discrimination is not sufficiently compelling enough of a purpose

Court held that providing Black role models is not a sufficiently compelling government purpose

Richmond v. J.A. Croson[edit | edit source]

Facts: The City of Richmond adopted the MBUP that required primary contractors to whom the city awarded construction contracts to subcontract at least 30% of the dollar value of the contract to one or more Minority Business Enterprise. The 30% set-aside did not apply to primary contractors that were themselves controlled by minority groups.

Rules: Without evidence of past particular race-based discrimination, a city may not enact a plan to provide a race-based set-aside to exclusively promote minority business enterprises, as this does not constitute narrowly tailored means geared towards accomplishing a compelling state purpose.

Reasoning: A state or local subdivision of government has the authority to eradicate the effects of private discrimination within its own legislative jurisdiction as long as it identifies such discrimination with sufficient particularity so as not to run afoul of the fourteenth amendment.

Grutter v. Bollinger[edit | edit source]

Facts: The University of Michigan Law School followed an unofficial policy that sought to achieve student body diversity by giving substantial weight to the race of each applicant in making admissions decisions, in addition to its consideration of other academic and non-academic variables. (similar facts to Gratz)

Rules: Consideration of race as a factor in admissions by a state law school does not violate the fourteenth amendment because supporting student body diversity is a compelling state interest; however, the school must demonstrate it previously made serious, good faith consideration of workable, race-neutral alternatives to achieve the sought after racial diversity.

Reasoning: The Law School provides an individual, holistic review of each of its applicants and reasons that alternative methods of achieving the Law School’s purpose risk sacrificing both academic excellence and other types of diversity in the school. However, the Law School should cease racial consideration in its admissions policies after instances of past discrimination have been sufficiently remedied.

Gratz v. Bollinger[edit | edit source]

Facts: Gratz and Hamacher were both white people that applied for admission in to the University of Michigan’s undergrad program. Both were denied admission and filed suit in federal district court against a university administrator, seeking to challenge the university’s admission policy on the grounds that it violated the equal protection clause of the fourteenth amendment. The challenged policy ranked applicants on a 150 point scale that accorded different points to factor such as grade, tests, and achievements. However, an applicant automatically received twenty bonus points for being an underrepresented minority. D

Rule: A university’s admissions policy that automatically gives preference to minority students on the basis of race, without additional individualized consideration, violates the Equal Protection Clause of the Fourteenth Amendment.

Reasoning: The university’s use of race as justification for automatically assigning twenty points to each minority applicant is not narrowly-tailored to achieve its purpose of promoting student body diversity. Under this system, applicants are not afforded individualized review and the extra twenty points virtually guarantee admission to any minimally qualified minority applicant.

Fisher v. University of TX at Austin[edit | edit source]

Top 10% plan for ¾ of the class; rest filled based on high school GPA, test scores, essays, and race

A court must engage in strict scrutiny of a public university’s race-based admissions process and afford no deference to the university’s chosen method of considering race to promote diversity

Schuette v. Coalition to Defend Affirmative Action[edit | edit source]

Court held that a Michigan initiative that amended the state constitution to prohibit the state or any of its political subdivisions from discriminating against or giving preference to any person, on the basis of race or gender, in education, contracting, or employment, did not violate the equal protection clause.

No state is required affirmative action and a state may choose to eliminate it.

Parents Involved in Community Schools v Seattle School District No. 1[edit | edit source]

Facts: Seattle school district No. 1 and Jefferson County School district voluntarily adopted student assignments plans that relied on race to determine which public schools certain children could attend. In each case, the schools used this system to ensure that the racial balance in any given public school fell within a predetermined range based on the racial composition of the school district as a whole. Parents Involved in Community Schools (PICS) (plaintiff) were parents of students denied assignment to particular schools under these plans solely because of their race.

Rules: Public schools may not assign students to schools solely on the basis of race for the purpose of achieving racial integration, although the use of narrowly tailored, race-conscious objectives to achieve general diversity in schools is permissible.

Reasoning: School assignments relying on racial classifications are subject to strict scrutiny. The school district’s use of racial classifications must be narrowly tailored to achieve a compelling government interest. There are two government interests that qualify as compelling: the interest of remedying the effects of past discrimination; and the interest of promoting student body diversity in the context of higher education. The racial assignment programs at issue are not related to either of this previously recognized interest. The school districts cite studies showing that students tend to gain intangible benefits from being educated in a racially diverse environment. This argument is rejected. The school districts provide no evidence that the amount of racial diversity necessary to achieve these intangible benefits coincides with the amount of racial diversity achieved by their racial assignment policies.

Other Discrimination[edit | edit source]
Discrimination against non-citizens[edit | edit source]

This is different from national origin-based discrimination

Non-citizens are covered by the text of the equal protection clause

Strict scrutiny is used here generally, except in cases of “alienage” where a rational basis test is used instead.

Graham v. Richardson[edit | edit source]

Facts: Arizona participated in an assistance program funded in part by federal grants and administered by Arizona under federal guidelines. In addition to the federal requirements, Arizona required that program beneficiaries be US citizens ore have resided in the US for at least 15 years. Similarly, PA funded its own welfare program, but only US citizens were permitted to receive benefits. Richardson was a lawfully admitted resident alien but was denied welfare because she did not meet Arizona’s residency requirement. Leger was a lawfully admitted resident alien denied in PA. Both filed separate class actions against Graham, the Commissioner of AZ’s public welfare department and PA’s welfare department. Both alleged that the requirements violated the equal protection clause.

Rules: Under the equal protection clause, states may not include conditions to the receipt of welfare benefits based on the beneficiary’s US citizenship or years of residence.

Reasoning: The plaintiffs were entitled to receive the same benefits as any US citizen. The guarantee of equal protection of laws to all persons necessarily include lawful resident aliens. State “classifications based on alienage, like those based on nationality or race, are inherently suspect and subject to close judicial scrutiny.” Aliens are a “discrete and insular” minority and states will only be able to regulate aliens as a class in a narrow circumstance.

Some classifications can be upheld only if they are necessary to accomplish a compelling government purpose. Further, the federal government has plenary power over matters of immigration and naturalization, and Congress has not imposed burdens on aliens who become indigent after admission. Although there is no need to decide whether aliens have a right to travel, once admitted, they are free to live in any state on equal privileges with all citizens under non-discrimination laws.

Foley v. Connelie[edit | edit source]

Facts: Under NY state law, no person shall be appointed to the state police force unless they are a citizen of the US. Edmund Foley was a lawfully admitted permanent resident in the US and was eligible for naturalization. He applied for admission to be a New York state trooper; a position conditioned on the passage of several competitive examinations. However, New York state officials prohibited Foley from taking the Police Department Examinations because Foley was not a citizen.

Rules: A state may confine employment in its police force to US citizens only because cops perform basic governmental functions that may be constitutionally reserved for members of the national political community.

Reasoning: Immigrants enjoy many freedoms in the United States that are not presented in other nations. However, no precedent exists that immigrants are entitled to unlimited freedoms in the United Sates or that all limitations are automatically suspect and subjected to strict scrutiny. Such a holding would depreciate the historic values of citizenship and jeopardize the existence of a strong political community in the United Sates. A lesser standard of scrutiny is required here. For an alienage classification in a connotationally state-controlled matter to be upheld, a state must justify the classification through a showing of some rational relationship between the interest protected and the limiting classification. The US has an interest in its political community for example, it may require government officers involved in the making of state policy to be US citizens. Although cops do not formulate laws, they are endowed with a great amount of discretion for carrying out those laws. Hence, states can constitutionally limit the participation of aliens in their police departments.

Ambach v. Norwick[edit | edit source]

Facts: Under NY Education law section 3001, a person who is not a citizen of the US may not be certified as a public school teacher unless they manifest an intention to apply for a citizenship. Norwick was born in Scotland and was a subject of Great Britain. She had resided in the US since the 60s and married a US citizens. Dachinger was a Finnish subject who came to the US also in the 60s and also married a US citizen. Both met the educational requirements to be public school teachers in NY. However, both refused to seek citizenship despite their eligibility to do so, and as a result, were denied certification as public school teachers for failure to meet the requirements of NY Education law section 3001.

Rules: A state may refuse to employ as elementary and secondary school teachers aliens who refuse to seek naturalization.

Reasoning: A rational basis test is used here. This distinction is based on the fundamental importance the Constitution places on citizenship. In determining whether teaching in public schools constitute a government function for equal protection purposes, the Court should look to the role of public education and to the degree of responsibility and discretion teachers have in fulfilling that role. Here public education fulfills an extremely fundamental role in society that it prepares people for participation as citizens in a representative government. Teachers in the public school system are directly involved in preparing students for this purpose. Hence, teachers are important to carry out governmental functions in public education. As such, the Constitution requires only that a citizenship requirement applicable to teaching in public schools bear a rational relationship to state interest.

Here, the goal of educating children about government and helping them become responsible citizens constitutes a legitimate state interest. Section 3001(3) is rationally related to that interest because it excludes from the important role of teaching only those aliens that have demonstrated their unwillingness to obtain United States citizenship.

Congressionally Approved Discrimination[edit | edit source]
Matthews v Diaz –[edit | edit source]

The Court unanimously upheld a federal statute that denied Medicaid benefits to aliens unless they have been admitted for permanent residence and have resided for at least five years in the US. Here, the Court drew a distinction between alienage classification imposed by the federal government and those created by state and local governments. Strict scrutiny is used on the latter, but the Court said that the federal law was upheld because it was not wholly irrational and served a legitimate purpose of the federal government in preserving the fiscal integrity of the program.

However, in Hampton v Wong, the Court clarified this and articulated a distinction between decisions by Congress or the president and those by federal administrative agencies; ration basis review is used only for the former. The Court invalidated a federal civil service regulation that denied employment to aliens. The Court said that “if the rule were expressly mandated by the Congress or the President, we might presume that any interest which might rationally be served by the rule did in fact give rise to its adoption.” The Court therefore explained that if civil service regulations had been adopted via federal law or a presidential order, it would be justified by the national interest in providing an incentive for aliens to become naturalized.

Distinctions on the basis of citizenship that are contained in federal law and presidential orders get rational basis review

But not decisions by federal administrative agencies (Hamptom v. Wong)

Mathews v. Diaz - Difference is federal law vs. state law, and federal law gets rational basis review

Undocumented Aliens and Equal Protection[edit | edit source]
Plyler v. Doe[edit | edit source]

Facts: Taxes amended its education laws to withhold state funds for the education of children not legally admitted into the country. It also authorized school districts to refuse to enroll children not legally admitted into the country. Doe and other aliens affected by this amendment sued the state of Texas, alleging that the amended statutes were unconstitutional.

Issues: May states deny free public education to children not legally admitted in the US?

Rules: States may not

Reasoning: While the court must respect the political decision of Congress, especially in the area of immigration, states have no similar authority regarding the classification and discrimination of aliens. However, states do have the authority to take action regarding undocumented aliens if it complies with federal goals as well as furthering a legitimate state goal. There is no evidence of any federal policy that supports denial of education to undocumented children. A heightened level of judicial review should be applied in cases dealing with the children of undocumented aliens. Children of immigrants do not choose to enter the country unlawfully, and depriving them of an education will contribute to a large, disenfranchised underclass of undocumented aliens. Therefore, the law will only be held unconstitutional if it furthers a substantial goal of the state. There are three possible goals, yet none are substantial enough to validate the discrimination.

Gender Discrimination[edit | edit source]

The Supreme Court first addressed a gender discrimination issue in 1872 in Bradwell v Illinois which upheld an Illinois law that prohibited women from being licensed to practice law. A very short majority opinion ruled against Bradwell without considering gender discrimination.

The Emergence of Intermediate scrutiny[edit | edit source]

In Reed v Reed, the Supreme Court for the first time invalidated a gender classification, but the Court professed to apply only a rational basis review.

Frontiero v. Richardson[edit | edit source]

Facts: Congress passed a law granting members of the armed services with dependents an increased in housing allowance, as well as medical and dental benefits for their dependents. Under the law, a serviceman was permitted to claim his wife as a dependent, regardless of whether the wife was actually dependent. In contrast, a servicewoman was only permitted to claim her husband as a dependent based on a showing that her husband was actually dependent on the woman for more than half of his support.

Ruling: Governmental classifications based on sex are inherently suspect and must subjected to strict scrutiny.

Reasoning: Women, like racial minorities, have faced a long history of discrimination. As a result, women could not vote or serve on juries or bring suits in their own names, or own property or sometimes serve as legal guardians of their own kids. Although the plight of women has improved over time, women are still subject to discrimination in higher education institutions, the job market, and the political arena. Sex is an immutable characteristic determined solely by the accident of birth and discrimination on its basis has no relationship with one’s capabilities.

Craig v. Boren[edit | edit source]

Facts: An OK statute prohibited the sale of non-intoxicating 3.2 percent alcoholic beer to males under 21, but permitted the sale of such beer to women over age 18. Craig, a liquor vendor in OK sued against the OK state official, Boren, as the law violated the Equal protection act. The government alleges that the law was supposed to limit traffic accidents because statistically, young-men were apparently more likely to drink and drive.

Rules: A governmental regulation involving gender discrimination is constitutional if it is substantially related to the achievement of an important government purpose.

Reasoning: Although the government presented statistics to prove their point, they nonetheless fail to show the correlation with age and sex. Just because it showed that more males of the 18-20 age group were killed or arrested for drunk driving, it does not show that all that it was their sex that caused these accidents to happen. Likewise, the government failed to show how 3.2% alcohol is non-intoxicating.

United States v Virginia[edit | edit source]

Facts: The Virginia Military Institute (VMI) boasted a long and proud tradition as Virginia’s only exclusive male public college. The United States brought suit against Virginia and VMI alleging that the school’s male-only admission policy was unconstitutional insofar as it violated the Fourteenth Amendment’s equal protection clause. On appeal from a District Court ruling favoring VMI, the Fourth Circuit reversed. It found that VMI’s policy was unconstitutional. After which, VMI decided to create an alternative learning program for women that differed from VMI in its academic offerings, methods, and financial resource.

Rules: All governmental gender classifications must be substantially related to an important government purpose that can be demonstrated by the government if it offers and exceedingly persuasive justification for the classification.

Reasoning: The standard of review for any governmental gender classification is intermediate scrutiny. This standard requires the government to provide an exceedingly persuasive justification for policies that discriminate against women. Inherent differences exist between the binary sexes, but these differences can be used only for the purpose of remedying the history of discrimination against women and not for the denigration of members of either sex. Virginia has not shown an exceedingly persuasive justification for excluding all women from VMI’s training. Virginia argues that the existence of a single-sex school furthers the important purpose of maintaining diversity in public school education, this argument is rejected. Had it really wanted to promote diversity, it would not have discriminated against women in the first place.

When is it discrimination?[edit | edit source]

To prove if there is gender discrimination, the two factors must be met

The gender classification can exist on the law. That is, the law in its very terms draws a distinction among people based on gender.

If a law is facially gender neutral, proving a gender classification requires demonstrating that there is both a discriminatory impact to the law and a discriminatory purpose behind it.

Geduldig v. Aiello[edit | edit source]

Facts: The State of California administered a disability insurance system that paid benefits to people in private employment who were temporarily unable to work because of a disability not covered by worker’s comp. The program received no state funding but instead was funded entirely by contributions of one percent of the wages of the participating employees. Aiello and other women suffered disabilities from pregnancies, sued the director of California’s Department of HR Development, Geduldig. They allege that the law discriminates against women because it does not include disabilities resulting from pregnancies.

Rules: Discrimination based on pregnancy in a state disability insurance program is subject to rational basis review.

Reasoning: Such discrimination does not violate the Equal Protection Clause’s prohibition on gender discrimination. One of the main benefits of the CA program is the fact that it is completely self-sustainable and does not rely on state funding. However, if CA extends its coverage to all temporary disabilities related to pregnancies, then the program could not run on contributions alone. Thus CA chose to focus more on employment related disabilities, but only some and not all.

Gender Classification Benefiting Women[edit | edit source]

Interestingly, the majority of supreme court cases concerning gender discrimination have involved laws that benefit women and disadvantage men. Two principles emerged from these decisions.

Gender classifications benefiting women based on role stereotypes generally are not allowed

Gender classifications benefiting women designed to remedy past discrimination are generally permitted.

Orr v. Orr[edit | edit source]

Facts: William and Lilian Orr were divorced in 1974. Under Alabama’s alimony law, husbands, not wives, are required to pay alimony upon divorce. As a result, William was required to pay monthly alimony to Lilian. Lilian sued William for nonpayment.

Rule: A state alimony law may not discriminate on the basis of gender if the state’s compensatory and ameliorative purpose are equally served by a gender-neutral classification.

Reasoning: If the purpose of the alimony statute is to announce the state’s preference for an allocation of family responsibilities in which the wife plays a dependent role, then it is unconstitutional because that is not a legitimate purpose. In contrast, Alabama states two other purposes for the law 1. To help needy spouses and 2 to compensate women for past discrimination in marriage and divorce. Requiring men, and not women to pay alimony is not substantially related to either of the stated purposes. Alabama already conducts individualized administrative hearings as part of each alimony ruling. The hearing officer can easily consider the parties’ financial status and assign alimony on is better equipped to pay. Hence, there is no need for the state to generalized based on gender.

Mississippi University for Women v. Hogan[edit | edit source]

Facts: The Mississippi University for Women (MUW) was a state-sponsored single-sex public university. MUW upheld a policy of excluding men from admission. Joe Hogan was an RN in Mississippi that did not hold a degree in nursing. However, he had worked for over five years as a nursing supervisor in Columbus, the city where MUW is located. He applied for the MUW’s nursing program and was denied based o his gender despite being qualified.

Rules: A statute that discriminates on the basis of gender may be unconstitutional if the statutory objective itself reflects archaic and stereotypical notions relating to gender.

Reasoning: For a state law that discriminates based on gender to be upheld, the proponent of the law must demonstrate an exceedingly persuasive justification” for the discrimination. This burden is met only be showing that the discriminating classification serves an important governmental objective and that the discrimination is substantially related to the achievement of the objective. However, when reviewing the constitutionality of a law, the reviewing court must also take care in ascertaining whether the statutory objective itself reflects archaic and stereotypic notions relating to gender. Here, the school states that it needed to remedy past discrimination. However, the court must still check the policy as a whole to see if whet her the actual purpose is based on gender stereotypes. The school did not have a legitimate reason for its policy. A state can use the purpose of compensating for past discrimination against a gender group only if the members of the gender that benefit by compensation are actually disadvantaged before through discrimination. Here, the state failed to show when women were discriminated from entering nursing schools. Additionally, the female-only policy reflects archaic stereotypes.

Michael M. v. Superior Court of Sonoma County[edit | edit source]

Facts: A California law defined statutory rape as sexual intercourse with a female under the age of 18 who is not the perpetrator’s wife. Michael M was a 17 year old male charged with statutorily raping a 16-year old girl. He sought dismissal of his charges on the grounds that the law violated the Equal Protection Clause. The trial court denied his motion and the Supreme Court of California affirmed. The state supreme court found that the statutory rape statute discriminated on the basis of sex because only women can be victims and only men can be charged with violating it. The state claims that the law was to prevent teen pregnancies.

Rules: A state statutory rape law that discriminates against males does not violate the Equal Protection Clause because it deters men from engaging in sexual behavior that might lead to illegitimate pregnancies.

Reasoning: The statute stated purpose in enacting the law was preventing illegitimate teen pregnancies. This finding by the legislature is entitled to great deference. The state of California has a significant interest in preventing illegitimate pregnancies among teen females because of the significant physical, emotional, and financial risk that comes with the pregnancies. When a teen is pregnant, certain complications arise that can only be borne by teen girls. Additionally, very few similar risks exist to deter teen males from engaging in sexual activity that leads to pregnancies. The state acted rationally in creating this law.

Rostker v. Goldberg[edit | edit source]

Facts: When Russia invaded Afghanistan in the 80’s, president Carter reactivated the draft registration process. Carter attempted to expand the scope of the draft by requiring both males and females to register. Congress reacted by passing the Military Selective Service Act which prohibited the president from requirement women from registering. Goldberg, a male citizen, and several other males brought suit against Rostker.

Rules: A congressional act that requires men and not women to register for a military draft does not violate the due process clause because women cannot statutorily participate in combat and thus are not similarly situated as men.

Reasoning: The Court is required to give special deference to decisions of Congress in the realm of national defense and military affairs. Congress’s goal in passing the MSSA of “raising and supporting armies” is an important government interest. Congress passed the MSSA to provide a mechanism by which armies can be raised to respond quickly to a combat situation. Congress’s purpose in enacting the MSSA is to facilitate combat. However, all branches of the military have enacted statutory prohibitions on the participation of women in combat. There is no reason to require women to register for a draft that is designed to address an emergency combat situation that they are not prepared to handle.

Gender Classification Benefiting Women as a Remedy[edit | edit source]

Califano v Webster[edit | edit source]

Facts: Under section 215 of the Social Security Act, old-age insurance benefits were computed based on a wage-earner’s average monthly wage minus his or her lowest years of earnings during a specific period of employment. Until a 1972 amendment to the SSA, this period of employment was treated differently for males and females. Webster (plaintiff) brought suit against Califano (defendant), administrator of the Social Security program, in federal district court on the grounds that the statutory scheme privileging women was unconstitutional.

Rules: A social security act provision that advantages women in calculating old-age insurance benefits does not violate the equal protection clause if it is directed towards remedying the historic effects of economic discrimination against women.

Reasoning: All governmental classification based on gender must serve important governmental objectives and must be substantially related to the achievement of those objectives to past Constitutional muster. Traditionally, the reduction of the disparity in economic conditions between men and women have been recognized by the Court as one such important governmental objective. Congress did not pass the SSA based on archaic and stereotypical generalizations about women, rather, the sole purpose of the SSA is to attempt to correct the economic disparities historically facing women in the job market.

Nguyen v. Immigration and Naturalization Service[edit | edit source]

Facts: A federal law stated that a child born out of wedlock in a foreign country to an American mother was automatically an American citizen. However, the law stated that a child born out of wedlock in a foreign country to an American father was not automatically an American citizen. Under the latter situation, the law provided that the child could be come a citizen if the father established paternity before the child turned 18. Nguyen was born in Vietnam to a Vietnamese mother and American father. Nguyen became a permanent resident of the US when he was six. At 22, Nguyen pled guilty to sexual assault on a minor. The INS commenced a deportation proceeding. Boulais, the father, did not establish that he was Nguyen’s father until Nguyen was 28.

Rules: A federal law establishing different requirements for derivative citizenship of an illegitimate child born abroad based on the citizen parent’s gender does not violate the equal protection clause.

Reasoning: The statutory citizenship requirements represents a governmental gender clasffication. The law must be evaluated under an intermediate scrutiny standard to see if the requirements are substantially related to accomplishing an important government interest. The statute requires one of three affirmative steps to be taken by a child seeking American citizenship when the child’s citizen parent is the father, but not if the citizen parent is the mother: legitimation; a declaration of paternity under oath by the father; or a court order of paternity. Congress rationally chose to impose these requirements on unmarried men and not unmarried women because of significant biological differences.

Specifically, the imposition of strict proof requirements for a paternal relationship and not a maternal one is justified by two government interest

The interest that a parent-child relationship exists

The assurance that the child and the citizen parent have some demonstrated opportunity or potential to develop not just a legal relationship, but an actual bond.

The Court can readily see a mother-child relationship just by her giving birth; however, this opportunity does exist for unmarried fathers who are not always aware that a child was conceived.

Other discriminations.[edit | edit source]

Rational basis review

Discrimination on the basis of age

Discrimination on the basis of mental or physical health (but remember Cleburne)

Discrimination on the basis of wealth or poverty

Discrimination on the basis of sexual orientation

SOUTH BAY UNITED PENTECOSTAL CHURCH, ET AL., v. GAVIN NEWSOM, GOVERNOR OF CALIFORNIA[edit | edit source]

The Issue: California has enacted a prohibition that may have unfairly targeted or disadvantaged religious gatherings. The prohibition prevents singing indoors during the Covid-19 pandemic.

The Guidelines said “Places of worship should discourage audience members from singing, chanting, and similar practices that may increase the likelihood of transmission from contaminated exhaled droplets and aerosols. Performers who are singing, chanting, or playing wind instruments without masks must follow the guidance for live events and performances.”

Chief Justice Roberts Concurrence: Roberts acknowledges the likely background, competence, and expertise politically accountable officials have to asses public health. However, the state has concluded that singing indoors poses a great risk of transmitting Covid. Roberts sees no basis in this record for overriding that aspect of the state public health framework. However, at the same time, the state presents determinations – that the maximum number of adherents who can safely worship in the most cavernous cathedral is zero – appears to reflect not expertise or discretion, but instead insufficient appreciation or consideration for the interests at stake.

Statement of JUSTICE GORSUCH, with whom JUSTICE THOMAS and JUSTICE ALITO join:

Often, Courts addressing free exercise challenge face the question of whether a law reflects “subtle departures from neutrality,” “religious gerrymandering,” or “impermissible targeting” of religion.” But not here. Since the start of Covid-19, California has openly imposed strict rules on religious organizations more than any other business.

Apparently, California is the only State in the country that has gone so far as to ban all religious activities. When a state obviously targets religion for differential treatment, the justices job becomes much clearer.

The prohibition claims to have the compelling government interest of public health. Gorsuch acknowledges that they are not scientists or experts in public health, but the Court may not abandon the field when government officials with experts in tow seek to infringe on Constitutional Liberties. The whole point of strict scrutiny is to test the government’s assertion, and the Court’s precedents make plain that it has always been a demanding, and rarely satisfied standard. Even in times of crises, especially in times of crises, the Court has a duty to hold governments to the Constitution.

Still, California says it can thread the needle. It insists that religious worship is so different that it demands especially onerous regulations. The state offers essentially four reasons why

Religion involves a large number of people mixing from different households

…. In close proximity

…. For extended periods of time

…. With singing.

The Court is not downplaying the suffering many have experienced during Covid. But California errs to the extent that it suggests that these four factors are always present in worship or have never been present in secular activities.

Large amount of people

Here, the government made no difference between worship and people waiting in check outlines or being pack into trains. It also doesn’t consider small group meetings for worship or confession. Nor does California explain why the less restrictive option of limiting the number of people who may gather at one time is insufficient for houses of worship, even though it has found that answer adequate for so many stores and businesses.

Close interaction

The government tours the mild climate in suggesting that worshippers might enjoy more space outdoors. Yet, California is not as concerned with the close proximity of hairstylist or manicurists to their customers. The state also does not force them or retailers to do their businesses in parks or lots. Nor does it explain why the narrower option it considers in many secular settings such as social distancing, masks, plexiglass, and cleaning, does not suffice for worship.

Time

California worries that worship brings people together for an extended period of time. Yet, it does not limit citizens from running in and out of other establishments; no one is barred from lingering in malls, salons, bus terminals. Nor has the government explained why narrowly tailored options , like reasonable limit on the length of religious gatherings, would fail to meet its concerns.

When it comes to the first three factors, California singles out religion for worse treatment than many secular activities. At the same time, the State fails to explain why narrower options it finds sufficient in secular context do not satisfy legitimate state interests.

Singing

California has sensibly expressed concerns that singing may be particularly potent in transmitting covid and, thus it banned singing at all indoor private gatherings. But, on further inspection, the singing ban may not be what it appears. It seems that California’s powerfully entertainment industry is exempted from this. So, once more, we appear to have a State playing favorites during a pandemic, expending considerable effort to protect its lucrative industry. Like earlier, California failed to explain why even a single masked cantor cannot lead worship behind a mask or plexiglass. Or why even a lone muezzin may not sing the call to prayer from a remote location inside a mosque.

California has been moving the goalpost for its “temporary” ban on indoor worship, however, it can’t hardly wait for its lucrative movie studios, malls, and cosmetologists. Drafting narrowly tailored regulations can be difficult. But if Hollywood may host a studio audience or film a singing competition, while a single soul may not enter a place of worship, something has gone awry.

Justice Barret and Kavanaugh Concurring: (Both agree with the statement made by Justice Gorsuch) As the case appears, it remains unclear whether the singing ban applies across the board (Generally applicable) or favors certain sectors. Of course, if a chorister can sing in a Hollywood studio, but not the church, then the regulation is not neutral.

JUSTICE KAGAN, with whom JUSTICE BREYER and JUSTICE SOTOMAYOR join, dissenting:

Justices of this court are not scientists. Nor do they know much about public health policy. Yet today, the Court displaces the judgements of experts about how to respond to a pandemic. The Court orders California to weaken its restrictions on public gatherings by making a special exception for worship services. The majority does so even though the policies treat worship just as favorably as secular activities that, according to medical experts, pose the same risk of Covid transmission.

The governing caselaw demands neutrality for free exercise. A government cannot put limits on religious activities if fails to prohibit nonreligious activities that endangers the government’s interests in a similar or greater degree. The Constitution does not require things which are different in fact to be treated in law as they were the same. States must treat like cases alike, but dissimilar cases accordingly.

The state has put limits on areas like lecture halls, theaters, and even churches. The scheme hones in on these indoor gatherings because they pose a heightened danger of Covid transmission. The Department of Public Health explained that there is a broad consensus among medical experts that transmission of the novel coronavirus is more likely at indoor public gatherings which bring together many different people from different households. To further elaborate on this point, experts says that increase in risk when gatherings are of an extended duration and when there is a lot of verbal interaction. That risk, of course, extends not only to participants, but everyone they associate with in the community.

The state regulation is less stringent on  shopping malls because they involve less proximity and less time than indoor worship or similar events do. Shoppers are less likely to receive a sufficient viral load of droplets to contract COVID. The State has regulated religious conduct with non religious conduct in a similar way. The only secular conduct that the State treats better is the kind that experts have found does not imperil its interests – the kind that poses less risk of Covid transmission.

ROMAN CATHOLIC DIOCESE OF BROOKLYN,NEW YORK v. ANDREW M. CUOMO, GOVERNOR OF NEW YORK[edit | edit source]

The Issue:  There is an Executive Order that limits areas into only admitting 10 and 25 people inside. Two organizations, one a Catholic Church, and another a synagogue, filed this application against the Executive Order issued by the Governor of New York.

There are two classifications of areas, red and orange. In red zones, no more than 10 people may attend each religious service, while in orange zones, it’s capped at 25.

The synagogue argues that the Governor specifically targeted Orthodox Jewish communities and gerrymandered the boundaries of red and orange zones. Both the church and synagogue maintain that the regulation treat houses of worships much more harshly than comparable secular facilities. And, they add that without contradiction, they have complied with all public health guidance, have implemented additional precautionary measures, and have operated at 25% or 33% capacity for months without a single outbreak.

Likelihood of Success on the Merits – The applicants have made a strong showing that the challenged restrictions violate the minimum requirement of neutrality to religion.

In the red zone, while a synagogue or church may not admit more than 10 people, businesses labelled as essential may admit whoever they wish. This list of essential businesses include may establishments not essential, like acupuncturists, camp grounds, and garages. The disparate treatment is even more striking in an orange zone. While attendance is limited to 25, non-essential businesses are allowed to admit whoever and no matter how many they want. The results are troubling as the District Court mentions, there could literally be hundreds of people shopping in malls on any given day, yet a nearby place of worship would be prohibited to 10 or 25. The Governor has stated that factories and schools have contributed to the spread of Covid, but they are treated less harshly than the church.

Although public health is compelling state interest, it is hard to see how the challenged regulations can be regarded as narrowly tailored. They are far more restrictive than any Covid-related regulation that have previously come before the court, more so than any other jurisdiction hard-hit by covid. The District Court noted that there had not been any Covid outbreak in any of the Diocese’s churches since they reopened. Not only is there no evidence that the applicants have contributed to the spread of covid, but there are many less restrictive rules that could have been adopted. It is hard to believe that places of worship that can seat 400 to 1000 people would create a more serious risk than any other activity that the state allows.

Irreparable Harm – There can be no question that the challenged restrictions, if enforced, will cause irreparable harm. The loss of First Amendment freedoms, for even minimal periods of time, unquestionably constitutes an irreparable injury.

Although Catholics can just watch the mass on TV, they are barred from doing things that require in-person attendance, such as communion. Same goes for Jewish people whose sabbath requires attendance.

Public Interest – Finally, it has not been shown that granting the applications will harm the public. State has not claimed that attendance at the applicants’ services has resulted in the spread of disease. And the state has not shown that public health would be imperiled if less restrictive measures were imposed. Members of this Court are not public health experts, but even in a pandemic, the Constitution cannot be put away.

After the applicants filed this with the Court, the Governor has reclassified the areas. The dissenting opinion suggests to reject this application and let them file a new one again once there is a similar reclassification. But, there is no saying when it would get reclassified again. Not granting relief now would impair the rights of the two places of worship to exercise their religious beliefs.

First Amendment – Freedom of Religion[edit | edit source]

The first Amendment has the freedom to establish religion and the freedom to exercise religion, which are referred to as the establishment cause and free exercise clause. The Free exercise clause was applied to the states through its incorporation into the Due Process Clause of the Fourteenth Amendment. The framers did no entrust the liberty of religious beliefs to either clause alone. As the court declared in Lee v Welsman, a state-created orthodoxy puts at grave risk that freedom of belief and conscience which are the sole assurance that religious faith is real, not imposed.

The two clauses are at odds, after all, the government may be free to establish a national religion, but may not compel anyone who may want to exercise a different belief. The Court has recognized that this tension is inherent in the First Amendment, and has noted the difficulty in finding “a neutral course between the two Religion clauses, both of which are cast in absolute terms, either of which, if expanded to a logical extreme, would tend to clash with each other.

History in interpreting the religion clauses[edit | edit source]

As with all Constitutional provisions, some look to history as a guide to the meaning of the Religion Clauses. This is particularly difficult for those provisions because there is no apparent agreement among the framers as to what they meant. A too literal quest for advice from the framers on the issue is futile and misdirect.

  • Seperationism – secular government with strong division between church and state
    • Voluntarism – advancement of church should come from voluntary support not from support of the state
    • Seperatism – both religion and government function best if they are independent from each other
  • Non-prefernetialism
    • Don’t prefer one Christian sect over another
    • Don’t prefer one religion over another
    • Don’t prefer religion over non-religion.

What is Religion?[edit | edit source]

United States v Seeger[edit | edit source]

Facts: Seeger was convicted in the District Court of New York for refusing to submit into induction into the Armed Forces. He declared that he was conscientiously opposed to participating in the war for reasons of his religious beliefs, but preferred to leave the question of a supreme being open rather than a yes or no. But this did not necessarily mean that he lacked faith. He believed in devotion to goodness and virtue for their own sakes and religious faith is a purely ethical creed.

Rules: Because of how a large percentage of religions in the world derived their faith from more than just a supreme being, but rather through a belief of understanding and peace, and because many of these religions exist in America; the Military Act violated the Freedom to Exercise Clause by limiting its definition of religion to merely those relating to a supreme being.

Reasoning: There are a lot of religions in the United Sates, more than just those that believe in some sort of supreme being. Many religions and faits believe in an ultimate goal of peace and equality among all. Hinduism and Buddhism are beliefs that do not focus solely on a supreme being or at times, do not believe in one at all, but rather they believe in knowledge and self-purification. These religions exist in the United States, and it would be limiting to only recognize those with a God as being part of a religion.

Seeger and a later case, Welsh, did not offer any criteria for assessing whether a particular view is religious under this definition. However, the plurality explained that belief in God is characteristic of most religions, but not a prerequisite for religion.

Requirement for Sincerely Held Beliefs[edit | edit source]

United States v Ballard[edit | edit source]

Facts: The Ballards were indicted and convicted for using and conspiring to use the mail to defraud. They sent pamphlets and other literature for their “I Am” movement. They also started corporations and solicited memberships for this movement. The movement was supposedly about how the Ballards have the power to heal and other supernatural abilities.

Rules: The law knows no heresy, and is committed to the support of no dogma, the establishment of no sect. The first amendment not only forestalls compulsion by law or the acceptance of any creed or practice of any form of worship, it also safeguards the free exercise of the chosen religion.

Reasoning: The First Amendment embraces two concepts regarding religion; the freedom to believe and to act. A person is allowed to believe any religion and act on those beliefs without government interference. Different religions consider different things as heretical, heresy trials are foreign to the Constitution. If the jury were allowed to review the veracity of the defendants in this case, to see whether it’s heretical, then this would open the floodgates for other cases to question other religious practices. Additionally, the consideration of heresy would involve the government preferring one sect or religion over another to define what is heretical or not; which violates the religious freedoms in the first amendment.

Dogma[edit | edit source]

The Court have held in Thomas v Review Board of the Indiana Employment Security Division, that a person is guaranteed by the free exercise clause to believe in things that may not be shared by all members of their religious sect. It is not within the court’s purview to see whether the petitioner or his fellow worker more correctly perceive the commands common to their faith or not. Courts are not arbiters of scriptural interpretation.

Masterpiece Cakeshop, Ltd. v. Colorado Civil Rights Commission[edit | edit source]

Facts: The Mullins asked the cakeshop to make a cake to celebrate their same-sex wedding. The shop refused because the owner held sincere religious objections to same-sex marriage. He instead offered to sell generic cakes to the couple. The couple filed a complaint with the CCRC, triggering a multilevel review.

Rules: Adjudicatory proceedings against a person for unlawful discrimination must give neutral and respectful consideration to the person's defense of sincere religious motivation.

Reasoning: In protecting the civil rights of same sex couples, the government must also protect the rights of those who object on religious or philosophical grounds. Here, the commission three times upheld the baker’s right to refuse. By contrast, the commission ruled that he must make the cakes. The commission reached opposite results based on its determination of which baker’s position was reasonable or unreasonable rather than the sincerity of each of the baker’s position. The commission demonstrated hostility toward religion’s role in public life.

Laws before Division v Smith[edit | edit source]

The Supreme Court’s earliest treatment of free exercise of religion was in Reynolds v US. There, a federal law prohibited polygamy in the territories and a defendant argued that his Mormon religion allowed for many wives. The supreme court rejected the free exercise argument and claimed that the constitutional provision required an exemption from otherwise valid criminal laws. As a law of the organization of society under the exclusive dominion of the United States, it is provided that plural marriages shall not be allowed. Can a man excuse his practices to the contrary because of his religious belief? To permit this would make the professed doctrines superior to the law of the land, and in effect, permit every citizens to be a law unto themselves.

The Court drew a distinction between beliefs and action, the Free Exercise Clause limited government regulation of beliefs, but not action. Congress was deprived of all legislative power over mere opinion, but was left free to reach actions which were in violation of social duties or subversive to good order.

In Sherbert v Verner, the Court expressly held that strict scrutiny should be used in evaluating laws burdening free exercise of religion and declared the denial of employment benefits to a woman who was discharged from her job rather than work on her Saturday sabbath. The court explained “if, therefore, the decision of South Carolina is to withstand the appellant’s constitutional challenge, it must be either because her disqualification as a beneficiary represents no infringement of the State to her constitutional right to free exercise or because any incidental burden on free exercise may be justified by a compelling state interests.

Here, not only is it apparent that the appellant’s declared ineligibility for benefits derives solely from her religious practice, but the pressure on her to forego the practice is unmistakable. The ruling forces her to choose between following the precepts of her religion and forfeiting benefits, on the one hand, and abandoning one of the percepts to work, on the other. Governmental imposition of such a choice puts the same kind of burden on the free exercise of religion.

Compulsory Schooling[edit | edit source]

The only other case where the Court found a violation of Free Exercise during this time was in Wisconsin v Yoder, where the court held that free exercise of religion required that Amish parents be granted an exception from compulsory school laws for their teens because it is a historically known part of their religion that secondary to higher education would bring worldly influence on their children. These worldly influences are seen to conflict with their beliefs. The Court concluded that the self-sufficient nature of Amish society made education for teens unnecessary. The lack of two additional years of compulsory education would not impair the physical or mental health of their child or result in an inability to be self-supporting or to discharge the duties and responsibilities of citizenship or in any other way that would detract from the welfare of society.

Employment Division, Department of Human Resources of Oregon v. Smith[edit | edit source]

Facts: Oregon state law prohibits the knowing or intentional possession of controlled substances unless it is medically prescribed. Smith and Black were fired from their jobs after they ingested peyote for sacramental purposes at a Native American Church service. When they applied for unemployment benefits, the EDDHR said that they were ineligible for benefits because they have been discharged for misconduct.

Rules: Under the free exercise clause, a state may constitutionally refuse to carve out an exception from its generally applicable criminal laws for religious practices.

Reasoning: The first amendment protects the right to believe, but not the right to act. Here, the two argued they should be granted an exception. However, religious beliefs have never been held to excuse people from compliance with otherwise valid and neutral laws prohibiting conduct that the state is free to regulate.

Under strict scrutiny, the government must have a compelling state interest to burden religious practice. Here, the state’s interest to control the use of narcotics is compelling enough to warrant this burden.

Lukimi Babalu –[edit | edit source]

In that case, the Court struck down a law that prohibited ritual sacrifice of animals on the ground that it was not neutral or generally applicable. The wording was very much religiously aimed and its definitions did not prevent the slaughtering of other animals in other religious or animals for non-religious purposes. The government alleges that the law was made to prevent animal cruelty, but because of its lack of general application and neutrality, it fails to meet this interest.

Trinity Lutheran Church of Columbia, Inc. v. Comer[edit | edit source]

Facts: Missouri’s department of natural resources gives grant money to schools and non-profits to resurface playgrounds using rubber from recycled tires. The Church applied for a grant, but the department denied it solely because it was a church. The department’s policy categorically disqualified all religious organizations from the program because Missouri’s state constitution prohibits public funding of religion.

Rules: Excluding churches from a grant program that provides public funds to resurface playgrounds violates the Free Exercise Clause.

Reasoning: Disqualifying entities from receiving public benefits solely because of their religious character expressly discriminates on the basis of religion, triggering strict scrutiny. The Court recognized more than 50 years ago that denying or conditioning benefits based on religion may infringe on religious freedom. That means the policy expressly discriminates against churches because of their religious status, not because of what the church will do with the funds. Therefore, the policy infringes the free exercise of religion.

Statutory Protection of Religious Freedom[edit | edit source]

The religious freedom restoration act was adopted to negate the Smith test and require strict scrutiny for Free exercise claims. The act declares that its purpose is to restore the compelling interest test as set forth in Sherbert and Yoder and to guarantee its application in all cases where free exercise of religion is substantially burdened and to provide a claim or defense to people whose religious exercise is substantially burdened by the government.

Trying to get rid of Smith[edit | edit source]

Fulton v Philadelphia[edit | edit source]

Syllabus: Philadelphia contracts with private agencies, which certify prospective foster families under state criteria. However, Catholic Social Services refuse to certify unmarried couples or same-sex couplies. Other agencies do certify same-sex couples, and so no same sex couples would seek certification from CSS. The state informed CSS that unless it certifies same sex couples, it would no longer refer children to the agency.

CSS Filed suit.

The refusal of the state to contract with CSS unless CSS agree to certify same sex couples violates the Free Exercise Clause by requiring CSS either to curtail its own mission or to certify same sex couples as a violation of its religious beliefs. The state’s polices are neither neutral nor generally applicable so they are subject to strict scrutiny.

The contract’s non-discrimination requirement forbids interfering with the public accommodation opportunities of people based on sexual orientation. It defines public accommodation as “whose goods, services, facilities, privileges advantages or accommodations are extended, offered, sold or otherwise made available to the public. The Court held that certification as a foster parent is not readily accessible to the public.

The non-discrimination requirement of the City’s standard foster care contract is not generally applicable. The contract requires agencies to provide services defined in the contract to foster parents without regard to their sexual orientation. But, this section also permits exceptions to this requirement at the sole discretion of the commissioner. This inclusion for discretion renders the non-discrimination provisions not generally applicable.

A government policy can survive strict scrutiny only if it advances a compelling state interest and is narrowly tailored to achieve those interests. Philadelphia has no compelling interest in denying the CSS.

Opinion of the Court:[edit | edit source]

CSS sees marriage as being between a man and a woman, so it refuses to give referrals to same sex couples. It also sees adoption or parenthood as being something done by married couples; thus they refuse unmarried couples. However, although they deny these people, they do refer them to the many other foster care certification agencies in the state. CSS is protected by the First Amendment’s Free Exercise Clause, which states that Congress shall make no law that prohibits the free exercise of religion.

A law is not generally applicable it if invites the government to consider the particular reasons for a person’s conduct by providing a mechanism for individualized exceptions. Here, the City’s non-discrimination requirement has a clause where the commissioner can give exceptions based on their own discretions. Also, a lack of general applicability if it prohibits religious conduct but allows secular ones that undermine the government’s asserted interest in a similar way.

  • For example, in one case, Hialeah, there was an ordinance that prohibited animal sacrifices based on faith, the government said it was to ensure proper disposal of animal carcasses in the open and to protect public health; but it does not affect secular acts like hunters disposing their game anywhere in the streets.

The city said that it has no intention of granting an exception to CSS for religious reasons, but the City may not refuse to extend that exception to cases of religious hardships without compelling reasons. So it is not a generally applicable requirement.

The City says that “principles of neutrality and general applicability still constrains the government in its capacity as manager” but the Court have never suggested that the government may discriminate against religion when acting as a manager.

The City said that CSS conducted an unlawful public accommodations practice in violation of the fair practices ordinance. The ordinance forbids denying or interfering with the public accommodation opportunities of people or otherwise discriminating against them. However, foster care has never been treated as a public accommodation in the State. Certification as a foster parent is not readily accessible to the public because it involves a customized and selective assessment that bears little resemblance to staying in a hotel, eating at a restaurant, or riding a bus.

The City stated that it had a compelling reason; namely, to maximize the number of foster parents, protecting the city from liability, and ensuring equal treatment of prospective foster parents and foster children. The Court checks whether these interests are compelling enough to deny CSS an exception.

  • CSS is likely to increate the number of available foster parents.
  • The city offers only speculation of being sued over CSS’s practices
  • The City can grant exceptions in its own discretions, so it gives no reason why it cannot grant CSS an exception for religious reasons.
Concurrences[edit | edit source]

The three justices want to use this case to overturn Smith, several others made clear they are almost ready to do so.

  • Really go after Smith – argue Smith just made free exercise an anti-discrimination analysis.
  • Tie free exercise to “forbidding or hindering unrestrained religious practices or worship.” “The key point to present purposes is that the text of the Free Exercise Clause gives a specific group of people the right to do so without hinderance.”
  • The three justices suggest Smith should be replaced with a rule that “a law that imposes a substantial burden on religious exercise can be sustained only if it is narrowly tailored to serve a compelling government interest”
Gorusch Concurrence[edit | edit source]

The Court has failed to answer whether Smith should be overruled, and this maneuvering around the Smith may have long lasting effects on it should apply to different cases.

First, the City argues that its challenged rules qualify for that exception because they require all foster care agencies to recruit and certify same sex couples. For this part, the Majority assumes that Philadelphia’s rule is indeed neutral towards religion. The lower court noted that Philly’s FPO applies only to certain defined entities that qualify as public accommodations while the “generally applicable” law in Smith was applicable across the board. The Majority found that FPO was no general because it expanded the meaning of “public accommodations” not by using the FPO’s meaning, but by using a different law to define public accommodations.

The Majority argues that foster care fail to qualify as public accommodation because it involves a customized selective process; however, where did this come from? Not in the state statute nor any case law, nor the briefs. The Majority just made it up. It is wrong because other laws hold an illustrative list of public accommodations, which includes schools, which do have customized selection processes for its students and faculty.

The Majority feels like it’s reaching for anything to support is curious separate-statute move. Before concluding that a public accommodation law is generally applicable under Smith, courts must ask themselves whether it would be incongruous to apply that law to religious groups. The Majority doesn’t apply this nor does it answer it properly. The Majority says that the Rejection of Referral provision in the contract does not state a generally applicable rule because it contemplates exceptions. However, this applies to the referral stage of the foster process, where a kid is referred to a family; not when during recruitment or certification. For that stage, the contract prohibits discrimination all across the board.

Concurrence (Barret)[edit | edit source]

The Majority fails to answer whether to overrule Smith. The history is silent on the matter as there is no historical record that supports whether the framers understood the first amendment to require exemptions from generally applicable laws in at least some circumstances. The textual and structural arguments against Smith are compelling.

Alito Concurrence[edit | edit source]

The Alito concurrence is the same as the other two. (Just read Smith since it goes deeper on how Smith should be applied)

Competing Theories of the Establishment clause[edit | edit source]

Strict Separation[edit | edit source]

  • The first theory often is termed “strict separation.” This approach says that to the greatest extent possible, government and religion should be separated. The government should be, as much as possible, secular; religion should entirely be in the private realms of society.
  • There are problems with this approach. A complete prohibition of all government assistance to religion would threaten the free exercise of religion.

Neutrality Theory[edit | edit source]

  • A second major approach says that Government must be neutral on religion; that is, the government cannot favor religion over secularism or one religion over others.

Accommodation[edit | edit source]

  • A third major theory is termed an “accommodation” approach. Under this view, the Court should interpret the establishment clause to recognize the importance of religion in society and accommodate its presence in the government.

County of Allegheny v ACLU[edit | edit source]

Facts: This is about the Constitutionality of two recurring holiday displays located on public property in Pittsburgh. The first is a manger placed on the Grand Staircase of the courthouse. The second is a menorah placed outside the City county building.

Issues: Whether the placement of these two religious ornaments is sufficiently likely to be perceived by adherents of the controlling religion as an endorsement and by non-adherents as disapproval of their religion, and thus is unconstitutional.

Rules: Taken together with other ornaments represent both religious and secularized interpretation of winter holidays and thus are not an endorsement of a religion, but rather of the season.

Holding: No

Reasoning: The manger and menorah do send a religious message, the manger even more so, but it was placed along Santa’s house along with a talking wishing well. Likewise, the menorah does send a religious message, but it’s not a message most people would generally be familiar with unless they are members of those religions. The menorah was placed around other religious symbols like a Christmas tree and a saluting statue of lady liberty. Thus, taken together with their surrounding ornaments, it is clear that these displays are not endorsing either religion, but instead the holiday season.

Government Discrimination Among Other Religions[edit | edit source]

It is firmly established that the government violates the Establishment Clause if it discriminates among religious groups. Such discrimination will be allowed only if strict scrutiny is met.

Lemon v Kurtzman[edit | edit source]

Facts: These two appeals raise questions as to Pennsylvania and Rhode Island statutes providing state aid to church-related elementary and secondary schools. PA has adopted a statutory program that provides financial support to nonpublic elementary and secondary schools by way of reimbursement. RI has adopted a statute under which the State pays directly to teachers in nonpublic elementary schools to a supplement of 15% their annual salary.

Issues: Are these two statutes Constitutional under the Free exercise clause?

Rules: To consider whether the statute is unconstitutional under the free exercise clause, the following must be found (1) the statute must have a secular purpose; (2) its principal or primary effect must neither prohibit or advance religion; (3) the state must not foster an excessive government entanglement with religion

Holding: Yes

Reasoning: On their face, the statutes were made to look like they had a secular purpose. However, the cumulative impact of the statutes related with religion in an excessively entangling way with the government. In order to see whether the government entanglement with religion is excessive, we must examine the character and purpose of the institutions benefited and the nature of the aid that the state provides and the resulting relationship between government and religious authorities.

In particular, an extensive degree of state supervision and surveillance is necessary to ensure compliance with the statutory requirements separating secular and religious education.

The Requirement for a secular purpose[edit | edit source]

The first prong of the Lemon test is the requirement that there be a secular purpose for a law. For example, in Stone v Graham, the Supreme Court declared unconstitutional a state law that required the 10 commandments to be posted on the walls of every public school classroom because it had no secular purpose.

Requirement for a secular effect[edit | edit source]

The second prong requires that the principal or primary effect of a law must be one that neither advances or inhibits religion. In recent years, this often has been expressed in terms of symbolic endorsement.

Estate of Thornton v Caldor is an example where the Court used the second part of the Lemon test to invalidate a law. A Connecticut statute provided that no person may be required by an employer to work on his or her Sabbath. This was found unconstitutional because it favors people with religious interests over others.

Prohibition of Excessive Entanglement[edit | edit source]

The Final prong of the Lemon test forbids government actions that cause excessive entanglement with religion. The Court has said that a law violates the Establishment Clause when it requires a comprehensive, discriminating, and continuing state surveillance. The Court has also said that apart from any specific entanglement of the state in particular religious programs and assistance violates the Establishment clause if it carries the grave potential for entanglement in the broader sense of continuing political strife over aid to religion.

Engel v Vitale[edit | edit source]

Facts: The respondent board of education, acting in its official capacity under state law, directed the school district’s principal to cause the following prayer to be said aloud by each class in the presence of the teacher at the beginning of each school day “Almighty God, we acknowledge our dependence upon thee, and we beg thy blessings upon us, our parents, our teachers, and our country.” This daily procedure was adopted on the recommendation of the state Board of Regents as part of boosting moral and spiritual training in schools.

Rules: Under the establishment clause of the first amendment, state officials may not compel an official state prayer, even if the prayer is denominationally neutral and students have the option of remaining silent or being excused

Reasoning: The prayer is clearly a religious activity, and thus is unconstitutional for state officials, acting in their official capacity, to both compose and require a prayer. Historically, this very practice of establishing government composed prayers for religious services was one of the reasons that caused many of the early colonists to leave England and seek religious freedom in America. As such, the Constitution is particularly sensitive to preventing government action that officially establishes religion.

Lee v. Weisman[edit | edit source]

Facts: Robert E. Lee, a public middle school principal, invited a Jewish rabbi to say prayers at his school’s graduation ceremony. Daniel Weisman, whose daughter was among the expected graduates, sought a temporary restraining order in federal district court to prevent the rabbi from speaking at the graduation. His request was denied, and the rabbi delivered several prayers at the graduation. Weisman later sought a permanent injunction in federal court to bar this thing from happening again.

Rules: Under the Establishment Clause of the First Amendment, the government may not invite clergy to deliver prayers at a public school graduation ceremony.

Reasoning: At a minimum, the government may not coerce anyone to support or participate in religion or its exercise. Here, because students and parents are essentially obligated to attend the graduation, the recitation of prayers amounts to governmental coercion to participate in religious activities. The school principal, in choosing both the prayer and the rabbi, can fairly be said to have been acting on behalf of the state.

Santa Fe Independent School District v. Doe[edit | edit source]

Facts: Prior to 1995, the student elected to be the campus chaplain at Santa Fe High School customarily gave an admittedly “Christian” prayer over the stadium loudspeaker at each of the school’s varsity football game. Doe filed suit against the school on the grounds that permitting prayers before football games violated the free establishment clause.

Rules: Under the establishment clause of the first amendment, a public school may not permit student-led, student-initiated prayer at school sporting events

Reasoning: In authorizing the prayer, the government has not created a public forum because it permits only one student during the entire school year to access the forum for prayer. Rather, the government has created a limited public forum subject to all the limits and safeguards of the Constitution. Santa Fe’s policy is unconstitutional because it permits prayers offered by one student who was elected by a majority of students. Thus, only the majority’s religious viewpoint is represented, resulting in the endorsement of religion by the school district. Additionally, the text of the school district’s policy excessively entangles government with religion as it only permits students to say an invocation for the purposes of solemnizing an event.

Religion as a part of Government Activities: Religious Symbols on Government property[edit | edit source]

In a previous case, the Court considered the Constitutionality of a nativity scene and a menorah on government property. In two cases decided on the same day in 2005, the Court considered the constitutionality of Ten Commandments displays on government property.

The Court declared the display in McCreary County unconstitutional but upheld the display in Van Orden. In reading the decisions, consider whether there is a meaningful distinction between these cases.

McCreary County v. American Civil Liberties Union of Kentucky[edit | edit source]

Facts: The County offices in Kentucky posted versions of the Ten Commandments on the walls of their courthouses. In 1999, the ACLU sued the counties in federal district court on the grounds that the displays violated the Establishment clause. Before the district court ruled, the counties expanded their displays on two occasions and issued resolutions stating that the Ten Commandments were their precedent for legal code. They surrounded the Ten Commandments with additional, framed historical non-religious documents in what was called the “Foundations of American Law and Government Exhibit.” The district court ruled that all three displays had a religious purpose and thus violated the Establishment Clause, and court of appeals affirmed.

Rules: If a government action involving religion has an objectively observable religious purpose, then the action violates the First Amendment’s Establishment Clause.

Reasoning: The Establishment Clause requires government neutrality on religious issues. Although protecting religious freedoms means protecting religion generally, the government must act as neutral as possible about promoting religion over non-religion. If the government supports a religion, it may offend non-religious people, and religious leaders may end up getting more power. This fusion of church and state undermines liberty and social stability.

To avoid harm, the government must show that their message has a genuine secular purpose that is not secondary to a religious purpose. Based on Stone v Graham, the ten commandments are a primarily religious instrument but can be displayed in a secular way. It is the details of the government actions that matter.

Van Orden v Perry[edit | edit source]

Facts: In the Texas capitol building, there were markers commemorating the people, ideals and events that compose Texan identity. A part of that was a monolith with the 10 Commandments. However, there were also imageries like the eye of Horus, stars of David, and the Chi Rho.

Rules: Religious displays by governmental actors are constitutional if they predominantly convey historical and social meaning.

Reasoning: The decision would not be based on the Lemon test. The Court bases its decision on the nature of the monument and the country’s religious history. The 10 commandments had an undeniable religious significance. However, based on Texas’ history and government precedence, the display would be Constitutional.

American Legion v. American Humanist Association[edit | edit source]

Facts: After the great war, the military used the cross symbolically in military honors. The army marked soldiers’ graves with plain white wooden crosses or Stars of David instead of traditional rectangular slabs. Americans inextricably linked images of row upon row of white crosses with loves lost, making the plain white cross an iconic symbol. Ten fallen soldiers’ mothers and other residents of Maryland formed a committed in 1918 to erect a memorial and chose a cross. Later, the American Humanist Association sued claiming that the presence of the cross on public lands and its maintenance with public funds violated the first amendment’s establishment clause.

Rules: A presumption of Constitutionality applies to memorials displaying religious symbols on public land if they have stood for many years and are associated with historically secular purposes and traditions.

Reasoning: Lemon v Kurtzman gave a framework on how to handle establishment clause cases. The Lemon Test hinges on the purposes and effects of the challenged government action and the government’s entanglement with religion. But this test is problematic when applied to commemorative or celebratory purposes. Thus, a different approach must be taken.

Long standing monuments using religious symbolism rules

  1. The evidence of the original purpose is long gone
  2. The purpose for maintaining the monument and the message it conveys may be altered or expanded with time.
  3. Removing a long-standing religious symbol may be seen as “aggressively hostile to religion.”

For these reasons, the passage of time gives a strong presumption that a monument with religious symbolism.

The cross may have been a religious symbol, but the original purpose of the cross in this case has been long gone. The purpose of maintaining the monument has also been lost to time. Removing it, especially since its stance as a commemorative for soldiers, would be seen as an aggressively hostile to religion.

When can government give aid to religion?[edit | edit source]

Many establishment clause cases have involved the issue of government assistance to religion. Decisions in this area are numerous but often difficult to reconcile. The Court inevitable is involved in line drawing. Total government subsidy of churches or parochial schools undoubtedly would violate the Establishment Clause. Indeed, James Madison’s Memorial and Remonstrance Against Religious Assessments was made in the context of opposing a state tax aid to the church. But it also would be clearly unconstitutional if the government provided no public services to religious institutions.

Therefore, the Court must draw a line between aid that is permissible and that which is forbidden. No bright line test exists or likely ever will be developed.

Mitchell v. Helms[edit | edit source]

Facts: Chapter Two of the ECIA provides for the allocation of federal funds for educational materials and equipment, such as library materials and computers, to elementary and secondary schools. Funds are provided to both public and private schools, but private schools must demonstrate that the services, materials, and equipment they receive will be used for the purposes of implementing “secular, neutral, and non-ideological” programs. In Jefferson Parish, roughly 30% of the funds were allocated to private schools. Most of these schools were religiously affiliated.

Rules: A statutory scheme that allocates secular, neutral, and non-ideological educational materials to religious schools does not violate the Establishment Clause.

Reasoning: According to Agostini v Felton, the Court has developed a two-part test for this matter.

  1. Does the program or statute have a secular purpose?
  2. Does the statute or program have a primary effect of advancing or inhibiting religion?

Chapter 2 was given to religious schools without the consideration of their religious status. All schools, public and private are allotted Chapter 2 resources based on their enrollment. All materials that Chapter 2 funds provided are secular materials. If the schools were to use the funding for religious purposes, that is their own private decision that the government did not prevent nor endorse.

Zelman v. Simmons-Harris[edit | edit source]

Facts: The state of Ohio established the Pilot Project Scholarship Program to provide educational choices to families with children residing in the Cleveland City school district. The program was enacted to help combat serious problems with Cleveland public schools. The program provides tuition aids for students in k through grade 3, expanding each year through eighth grade, to attend a participating public or private school of their parents’ choosing. The program provides tutorial aid for students who choose to remain enrolled in public school. The tuition aspect of the program permits any private school, whether religious or nonreligious, to participate and accept program students. In 2000, 82% of participating private schools were religiously affiliated.    

Rules: The government can fund educational programs even when most students attend religious schools as long as there are nonreligious alternatives available.

Reasoning: The program was proper based on the Lemon test. The program had a secular purpose to fund schools. The primary effect of the program was neutral on religion, and the program did not create any excessive entanglement between the government and religion because the decisions to join religious schools were made by private individuals.